NCLEX

Pataasin ang iyong marka sa homework at exams ngayon gamit ang Quizwiz!

Name some foods that are rich in... A. Calcium B. Vit D C. Both Ca2+ and Vit D

A. - Cheese - Ice cream - Almonds - Green leafy vegetables (kale, spinach, broccoli) - Tofu B. - Canned tuna - Oily fish - Egg yolk - Cod liver oil C. - Milk - Yogurt - Salmon - Cereal - Canned sardines

A. What kind of needle must be used when withdrawing medication from a glass ampule? B. What are the steps in preparing medication from a glass ampule?

A. A filter needle Doing this will prevent aspiration and injection of glass shards. *A glass ampule is the type that you "snap" off/ have to break before drawing up the medication. B. 1. Flick upper stem with fingernail. 2. Use gauze to break neck AWAY from body. 3. Withdraw medication using a FILTER NEEDLE (you do not need to inject air!*) 4. Replace filter needle with injection needle. 5. Dispose ampule and filter needle in the sharps container.

Opioid withdrawal- A. What are the clinical manifestations? - GI - CV - Psych - Other B. What are the opioid agonists or nonopioids used for management of the withdrawal sx's?

A. GI: N/V, diarrhea, cramping, increased bowel sounds CV: increased HR/BP, diaphoresis Psych: insomnia, yawning, dysphoric mood (generalized dissatisfaction with life) Other: Myalgias (muscle pain), arthralgia (arthritic pain), lacrimation (tears), rhinorrhea (runny nose), piloerection (goose bumps), mydriasis (pupil DILATION) B. Opioid agonist: 1. Methadone (preferred) 2. Buprenophrine Nonopioid: CLonidine or adjunctive meds (antiemetics, antidiarrheals, benzos)

A. IV Magnesium sulfate toxicity when treating severe pre-eclampsia for seizure prophylaxis has what associated clinical manifestations? B. What is the tx for magnesium toxicity?

A. Mild: Nausea, flushing, HA, hyporeflexia Moderate: Arelexia, hypoCa2+, somnolence (sleepiness) Severe: Repiratory paralysis, cardiac arrest B. Stop Mag therapy and give IV calcium gluconate bolus

A. Pediculosis capitis 1. What kind of microorganism is it? B. Seborrheic dermatitis

A. "Head lice" 1. Parasitic infection (it is unnecessary to treat household pets with human lice) B. "Cradle cap"

*KNOW THIS- A. What is a normal Intracranial pressure (ICP)? B. What is a normal Cerebral perfusion pressure (CPP)? C. What is a normal mean arterial pressure (MAP)?

A. 0-15 mmHg B. 80-100 mmHg C. 70-105 mmHg *If the MAP falls below <60 mm Hg, vital organs may be underperfused and can become ischemic.

A. What is the therapeutic dose for lithium? B. What dose is considered toxic?

A. 0.6-1.2 mEq/L B. Doses over 1.5 mEq/L

What gauge IV is appropriate for... A. Blood products B. General IV fluids and drugs C. Rapid administration of IV fluids and drugs or for hypovolemic shock D. Children and some older adults with small, fragile veins Options are: 14G, 18G, 20G, 22G, and 24G

A. 18G B. 20-22G - A 20G is acceptable for a blood transfusion C. 14G D. 24G

Definitions of Term- A. Early term is considered? B. Full term is considered? C. Late term is considered? D. Postterm is considered? E. Late Preterm is considered?

A. 37-38 6/7 weeks B. 39-40 6/7 weeks C. 41-41 6/7 weeks D. 42 weeks and beyond E. 34-36 6/7 weeks

Hemophilia- A. What is it? B. What is the most frequent site effected (long-term complication)? C. What type of diet is indicated?

A. A bleeding disorder caused by a deficiency in coagulation proteins. When injured, clients with hemophilia should be monitored closely for external as well as internal bleeding. B. Joints, especially the knee causing destruction Over time, chronic swelling and deformity can occur. C. Regular diet

INR- A. What is it? B. What is the normal range? C. What risk comes with a higher INR? D. What is a normal PT range? E. What is a normal PTT range?

A. A blood test used to monitor the effectiveness of warfarin therapy (used with PT). B. INR normal range is 2-3 C. The higher the INR= the higher the bleeding risk D. Normal PT 11-12.5 seconds E. Normal PTT 60-70 seconds

Autism spectrum disorder- A. What is it? B. What are the clinical features/ 2 core sx's?

A. A complex neurodevelopment disorder characterized by the onset of abnormal functioning before age 3. B. 1. Social interactions and communication (verbal and nonverbal) 2. Patterns of behavior, interest, or activities that can be restricted and repetitive (Ex= exhibit same play routine with a single toy) AKA- these clients like structured routine and consistency during hospitalization.

Phenylketonuria- A. What is it? B. What are the consequences of no tx? C. What does the management include?

A. A metabolism d/o of the enzyme required to convert phenylalanine into the amino acid tyrosine. B. As unconverted phenylalanine accumulates, irreversible neurologic damage can occur. C. 1. Monitor phenylalnine serum levels 2. Include synthetic proteins and special formulas int he diet 3. Eliminate high-protein (high-phenylalanine) foods (ex= eggs, meats, and milk) from the diet 4. Encourage natural consumption of natural foods low in phenylalanine (most fruits and veggies)

Paracentesis- A. What is it? B. What are the nursing actions that you need to ensure during the procedure? (4)

A. A surgical puncture to remove fluid from the abdomen. B. 1. RN needs to verify that the client received necessary info to give consent and witness informed consent obtained by the HCP. 2. Instruct client to void before to prevent puncturing the bladder. 3. Assess client's abdominal girth, weight, and VS. 4. Place the client in HIGH fowlers position or as upright as possible.

Kawasaki dz- A. What is it? B. What are the clinical sx's? (*think "CRASH" and burn) C. What is the tx?

A. A type of vasculitis that causes blood vessel inflammation to the coronary arteries supplying blood to the heart. B. 1. Conjunctivitis 2. Rash on all body parts 3. Adenopathy (cervical lymph node enlargement) 4. STRAWBERY TONGUE 5. Hands and feet are swollen and have a rash 6. Fever for greater than 5 days with no resolution with antipyretics C. 1. IV IG (antibodies from other to reduce inflammation) 2. ASA/ Aspirin to prevent platelet aggregation

Norepinephrine (Levophed)- A. What is a serious consequence of this drug? B. How is it most likely to be administered? C. What is its antidote?

A. A vasoconstrictor and vesicant that can cause skin breakdown and/or NECROSIS if absorbed into the tissue. B. Should be infused through a CVC however it may be infused at lower concentrations via a large PIV for up to 12 hours until CVC access is established. C. Phentolamine

A. Pleural effusion B. What is the common treatment?

A. Abnormal accumulation of fluid in the pleural space surrounding the lungs. B. Thoracentesis - HCP will prepare the skin, inject a local anesthetic and then insert a needle between the ribs into the pleural space when the fluid is located.

A. Define atresia B. Define fistula

A. Absence or abnormal narrowing of an opening or passage in the body. B. An abnormal opening made passage between a hollow or tubular organ and the body surface, or between two hollow or tubular organs.

Colorectal cancer- A. What age does it occur most often? B. Symptoms include? (5)

A. Adults over age 50 B. 1. Blood in stool 2. Abdominal discomfort 3. Anemia (due to intestinal bleeding). 4. Change in bowel habits (due to obstruction by polyps or tumors). 5. Unexplained weight loss due to impaired nutrition from altered intestinal absorption.

Retinoblastoma- A. What is it? B. What is the most common clinical sx?

A. An eye cancer that begins in the back of the eye. B. Absence of the red eye reflex (when there is an absence of the reflex, there is a WHITE reflex on the abnormal eye from opacities of the lens [cataract] or tumor [retinoblastoma] while the other normal eye is red).

Mannitol- A. What is it? B. What is it used to treat? C. What do you as a RN need to look for? (what side effect?)

A. An osmotic diuretic B. 1. Cerebral edema (increased ICP) 2. Acute glaucoma C. When administered, Mannitol causes an increase in plasmaoncotic pressure that draws water into the intravascular space creating volume expansion. This fluid, along with the drug is excreted through the kidneys, thereby reducing the cerebral edema and increased ICP. - Higher doses of Mannitol may cause fluid overload that may cause life-threatening PULMONARY EDEMA (so monitor serum osmolarity, I&O, serum bytes, kidney function, and if crackles are noted in the lungs).

Sjogrne's syndrome- A. What is it? B. What is the tx?

A. Autoimmune condition - Cx's inflammation of exocrine glands (ex= lacrimal, salivary), resulting in decreased production of tears and saliva and leading to dry eyes and dry mouth B. OTC preservative-free artificial tears to relieve eye dryness, burning, itching, irritation, pain, and a gritty sensation in the eyes. - Artificial saliva is used to tx dry mouth and using a room humidifier

BMI Ranges- A. Underweight B. Normal weight C. Overweight D. Obese

A. BMI less than 18.5 B. BMI 18.5-24.9 C. BMI 25-29.9 D. BMI is 30 or more

Normal postpartum lochia A. Lochia rubra B. Lochia serosa C. Lochia alba

A. Birth to 3/4 days - dark or bright red blood; odor similar to menstrual blood; occasional small clots; decreases each day in quantity B. Serosanguineous (pink); brownish (old blood); quantity gradually decreasing in amount C. White/yellow color; creamy; light quantity

A. What does Failure to Thrive (FTT) indicate? B. Risk factors for FTT include? (11)

A. Characterized by a low weight/height ratio and/ or falling below the 5th percentile on the growth curve due to inadequate caloric intake, inadequate absorption of calories, or excess caloric expenditure B. 1. Young parent age 2. Unplanned/unwanted pregnancy 3. Lower levels of parental education 4. Single-parent home 5. Social isolation 6. Chronic life stresses/anxiety in the home 7. Disordered feeding techniques - Prolonged breast or bottle feeding - Unstructured meal times - Negatie or difficult interactions at meal time - Poor parental feeding skills - Negative attitudes toward food (fear of obesity or an overweight child) 8. Substance abuse 9. Domestic violence and/or parental history of child abuse 10. Poverty, food insecurity 11. Parents who have a negative perception of the child

Intussusception- A. What is it? B. What are the expected clinical findings?

A. Condition in which a part of the intestine folds into another leading to an obstruction. B. 1. Abdominal pain 2. "Currant jelly" stools 3. A sausage-shaped abdominal mass

Hydrocephalus- A. What is it? B. Tx

A. Condition in which excess CSF collects in the brain's ventricles, typically in young children; enlarging the head and sometimes can cause brain damage. B. Ventriculoperitoneal shunt (moves fluid from the ventricles of the brain to the abdominal cavity to reduce CSF excess)

A. Functional constipation implies? B. What is retentive encopresis?

A. Constipation that does not have a physiological cause. A person with functional constipation may be healthy, yet has difficulty defecating. B. Pooping pants WITH CONSTIPATION

Atelectasis- A. What kind of breaths sounds would you expect to hear? B. How do you tx?

A. Decreased breath sounds B. Incentive spirometry and deep breathing

A. What is dumping syndrome? B. How can you prevent dumping syndrome?

A. Delivery of a large amount of hyperosmolar chyme from the stomach into the small bowel. -Usually after vagotomy and a gastric drainage procedure (pyloroplasty/gastrojejunostomy) -Results in autonomic instability, abdominal pain, and diarrhea B. Client should eat multiple small meals, eat a LOW-carb diet, and separate their consumption of food and fluids.

A. How to draw up NPH and regular insulin B. What are the steps?

A. Draw up regular insulin first THEN NPH

Osteoporosis- A. What is it? B. What causes it? C. What hormones usually influence osteoblasts and clasts? (5) D. What are known risk factors? (7) E. S/S (5) F. How is it diagnosed? What do you need to ensure before the procedure? G. Nursing interventions (6) H. What are the medications used to tx? (5) I. What are common sites of fractures? J. What two ions are stored in the spongy bone?

A. Dz that thins the bones to a point that it cannot withstand normal stress resulting in bone fracture. B. Occurs when osteoclasts (breaking down bone) start to outwork the osteoblasts (building bone) C. 1. Parathyroid (releases PTH to increase bl Ca2+ levels) 2. Thyroid (releases Calcitonin to decrease bl Ca2+ levels) 3. Estrogen (reduced during menopause; usu controls blast and clast activity; secreted in the ovaries) 4. Testosterone (decreases with age; men are affected too) 5. Growth hm D. 1. Low intake of Ca2+ and Vit D 2. Age 30 years or older 3. Lifestyle (smoking, ETOH use, sedentary, immobile) 4. Caucasian or Asian; woman 5. Inherited/ hereditary 6. Under wt 7. Meds (taking glucocorticoids for greater than 3 mo; Anticonvulsants- Carbamazepine, Phenytoin, Phenobarbital, Vaproate) E. 1. Fractures (hips, spine, wrist) 2. Rounding of the upper back 3. Can be asymptomatic until a fracture occurs 4. Inches of height lost (2-3 inches from spinal fracture) 5. Low back, neck, hip pain with palpation or activity F. Bone density test (DEXA scan) - This is an X-ray imaging taken to measure Ca2+ and other minerals in bones - No presswork, but educate the patient to not take any Ca2+ supplements 24 hr prior to the scan (no tums, Rolaids, Vitamins) G. 1. Assess risk factors 2. Safety 3. Good body mechanics 4. PREVENTION: - Wt bearing exercise (wts, tennis, hiking) - Foods rich in Ca2+ (yogurt, milk, sardines, collard greens, broccoli) - Foods rich in Vit D (salmon, tuna, egg yolks, cheese) 5. Smoking cessation 6. Limit ETOH intake H. 1. Ca2+ and Vit D supplements 2. Biphosphonates (decrease bone breakdown) - Alendronate - Ibandronate - *Both of these cause GI upset and an inflamed esophagus= take with a full glass of water, an empty stomach, and sit up 30-60 min afterwards 3. Calcitonin - Increased risk for hypoCa2+ 4. Hormone Replacement Therapy (HRT)- Short term - Raloxifine 5. Teriparaticle I. Wrist, hips, and spine are common sites of fracture J. Ca2+ and phosphorus

Tonic-clonic seizures- A. What is it a form of? B. What is it classified by? (is it generalized or focal and sx's) C. What are safety precautions to implement when client is on seizure precautions?

A. Epilepsy (which means seizure d/o; recurring and unpredictable seizures= neurons are synchronously active from too much excitation with too little inhibition from GABA inhibitory neurotransmitter). B. A generalized seizure (means that both brain hemispheres are involved) AND 1. muscles suddenly tense up 2. then convulsions occur (muscles tensing then relaxing repeatedly) C. 1. Turn client on side 2. Provide O2 and suctioning as needed 3. Pad side rails or remove objects that are near the client to decrease the risk for injury during a seizure (avoid restraints)*

What is the antidote for... A. Benzodiazepines (like Midazolam)? B. Opioids (like Morphine; Oxycodone) C. Antidote used to tx a norepinephrine extravasation (when the drug leaks outside the vein onto or into the skin causing a reaction)? D. Benzotropine is used to treat?

A. Flumazenil B. Naloxone C. Phentolamine D. Extrapyramidal side effects

A client with hypothermia has just arrived in the emergency department via ambulance. The client is being rewarmed with blankets, and the IV fluids are being changed over to warmed fluids. What additional intervention is a priority? A. Attaching the cardiac monitor B. Covering the client's head C. Drawing blood for bytes and glucose D. Placing an additional large-bore IV catheter

Answer: A

A clinic nurse examines a client with a tentative diagnosis of primary Sjögren's syndrome. Which finding observed by the nurse would most likely be associated with this syndrome? A. Dry eyes and mouth B. Low back stiffness C. Multiple tender points D. Thickening of the skin

Answer: A

During the first prenatal assessment, the client reports the last normal menstrual period starting on March 1 and ending on March 5, but also slight spotting on March 23. The client had unprotected intercourse on March 15. Using Naegele's rule, what is the estimated date of birth? A. December 8 B. December 12 C. December 22 D. December 30

Answer: A

The HCP prescribes IV fluid resuscitation for a client in hypovolemic shock. The RN should anticipate the rapid infusion of which IV solution INITIALLY? A. 0.9% NS (or NS) B. 5% Albumin C. D5LR D. D5 and water

Answer: A

The nurse assesses a female client with a diagnosis of primary adrenal insufficiency (Addison disease). The nurse recognizes which finding associated with the disease? A. Bronze pigmentation of skin B. Increased body or facial hair C. Purple or red striae on the abdomen D. Supraclavicular fat pad

Answer: A

The nurse has received report on the following clients. Which client should the nurse assess first? A. Client 4 hr post-op colon resection who has a BP of 90/74 B. Client receiving palliative care who has Cheyne-Stokes respiration with 20-sec periods of apnea C. Client with anemia and Hgb of 7 who has a pulse of 110 after ambulation D. Client with DKA who has rapid, deep respirations at 32/min

Answer: A

The nurse is caring for a young adult who is considering becoming pregnant. The client expresses concern, stating, "One of my parents has Huntington disease, and I am afraid my child will get it." How should the nurse respond? A. "Genetic counseling is recommended. You will receive a referral before you leave" B. "Huntington dz inheritance requires both biological parents to carry the gene" C. "There are others ways to grow your family. You should consider adoption" D. "This dz occurs spontaneously and is not likely to affect your children"

Answer: A

The nurse notes documentation that a client is exhibiting Cheyne-Stokes respirations. On assessment of the client, the nurse should expect to note which finding? A. Rhythmic respirations with periods of apnea B. Regular rapid and deep, sustained respirations C. Totally irregular respiration in rhythm and depth D. Irregular respirations with pauses at the end of inspiration and expiration

Answer: A

The nurse cares for a client with type 1 diabetes mellitus who is obtunded and responding to only painful stimuli. A STAT blood sample reveals a blood glucose level of 38 mg/dL (2.11 mmol/L). Which initial action by the nurse is best? A. Admin 50% dextrose in water IVP B. Assist the client to drink 4 oz of orange juice C. Measure the client's HR and BP D. Observe for sweating, shakiness, and pallor

Answer: A Not B= b/c clients with an altered mental status are at high risk for aspiration and are not appropriate candidates for PO glucose replacement.

A new nurse is providing hospice care for a terminally ill client who reports dyspnea. Which intervention would cause the charge nurse to intervene? A. Admin O2 via non-rebreather mask B. Admin morphine PRN C. Providing a portable fan to improve air flow in the room D. Providing relaxation strategies such as music and guided imagery

Answer: A Think hospice, a nonrebreather mask is used to deliver high concentrations of O2 in EMERGENCY situations. Applying this mask may make the client uncomfortable, cause claustrophobia and increased anxiety. A low-flow O2 by nasal cannula should be provided instead.

The nurse reviews the serum laboratory results of a client who was seen in the clinic 2 days ago for worsening joint pain from a flare of systemic lupus erythematosus. Which result is of greatest concern and prompts the nurse to notify the health care provider? A. Cr of 1.8 B. Elevated erythrocyte sedimentation rate C. + antinuclear titer D. WBC of 3600

Answer: A (B/c normal is 0.6-1.3 mg/dL)

The nurse in the intensive care unit is caring for a client who is postoperative from a cardiac surgery. The client has a mediastinal chest tube. During assessment, the nurse notes bubbling in the suction control chamber. Which nursing action is appropriate? A. Assess the insertion site for presence of subcutaneous emphysema B. Notify the surgeon of a large air leak C. Take no action as the chest tube is functioning appropriately D. Turn down the wall suction until the bubbling disappears

Answer: C Not A= b/c this is air that has leaked into the tissue surrounding the chest tube insertion site. A crackling sensation is felt when palpating the skin. It does not affect bubbling within the chest tube drainage system. Not B= b/c an air leak would cause bubbling in the air leak gauge or water seal chamber not in the suction control chamber

The stages of fetal development- A. What is the time range in which the baby is considered an embryo? B. What happens at wk 4? C. What happens at wk 8? D. What is the time range in which the baby is considered a fetus? E. What happens at wk 12? F. What happens at wk 16? (3 important things*) G. What happens at wk 20? H. What happens at wk 24? I. What happens at wk 32?

A. From day 15 (week 2) to week 8 after contraception B. Heart starts to beat C. All organs are formed and placenta develops completely by week 8-10 D. Week 9- birth E. SEX is recognizable; fetus heartbeat identified using a doppler F. 1. Lanugo develops (fine downy hair that helps with determining maturity of the fetus) 2. Meconium develops in the intestines (black, tarry stool/ baby's first stool) 3. Quickening may be felt (spontaneous movements from the baby) G. 1. Vernix caseosa develops and lanugo covers entire body (its like a "lotion" that protects the baby's skin from watery damage of the amniotic fluid-- vernix decreases are you get closer to term) 2. Brown fat develops 3. Mom DEFINITELY should feel fetal movement 4. Heartbeat heard with fetoscope H. Surfactant develops that is needed for breathing at birth to help keep the alveoli open I. All body organs are fully developed, EXCEPT for the lungs

After the nurse receives the change-of-shift report, which client should the nurse assess first? A. Client with asthma who has SOB and high-pitched expiratory wheezing B. Client with DM and a stasis leg ulcer dressing saturated with serosanguineous drainage C. Client with HF who is SOB and coughing up pink frothy sputum D. Client with L pleural effusion and absent breath sounds in the L base

Answer: C Pink-frothy sputum= acute pulmonary edema which is a potentially life threatening condition

Gout- A. What is it? B. Who would likely be at risk of developing gout? C. What lifelong modifications would help prevent future exacerbations? 1. Achieve and maintain a healthy weight 2. Avoid foods containing protein 3. Drink plenty of fluids 4. Increase meat intake 5. Limit ETOH consumption

A. Inflammatory condition from ineffective metabolism of purines= uric acid accumulation in the blood B. Obese, HTN, Hyperlip, insulin resistance, poor diet, ETOH consumption, sedentary lifestyle C. 1, 3, and 5

A. What type of solutions are used for immediate fluid resuscitation in clients with hypovolemic shock? (Isotonic, hypotonic, or hypertonic) B. What type of solutions are typically used to treat hyperNa+?

A. Isotonic solutions (0.9% NS, LR) B. Hypotonic solutions (0.45% NS or D5W)

Viral croup syndromes- A. What is another name for this condition? B. What is croup? C. What is the cause? D. What are the common clinical manifestations? (1 really distinct one and 4 other manifestations that go along with it)

A. Laryngotracheobronchitis B. Inflammation of the larynx, trachea, and bronchi C. Parainfluenza, adenovirus, and respiratory syncytial virus (RSV) D. #1- Severe BARKING COUGH** 2. Stridor (noisy breathing) 3. Fever 4. Runny nose 5. Congestion

A. What is tine corporis? B. How does it spread? C. What are the preferred tx? D. What is the common sx?

A. Ringworm; a fungal infection on the superficial keratin layers of the skin, hair, and/or nails. B. Spread by contact and is highly contagious C. Antifungal cream (terbinafine/Lamisil) and/or medications (ex= miconazole) D. Itching

The nurse cares for a client with an established ascending colostomy. Which statement made by the client indicates that further teaching is required? A. "I always try to drink 3L of H2O each day" B. "I avoid eating beans, onions, broccoli, and cauliflower" C. "I change the appliance and bag every other day" D. "I empty the bag when its about 1/3 full"

Answer: C The appliance should be changed every 5-10 days.

When are each of the trimesters? A. First Trimester B. Second Trimester C. Third Trimester

A. Weeks 1-13 B. Weeks 14-26 C. Weeks 27-40

metabolic syndrome

AKA INULIN RESISTANCE SD** A syndrome marked by the presence of usually three or more of a group of factors (as high blood pressure, abdominal obesity, high triglyceride levels, low HDL levels, and high fasting levels of blood sugar) that are linked to increased risk of cardiovascular disease and Type 2 diabetes.

Which purposes of placental functioning should the nurse include in a prenatal class? Select all that apply. A. It cushions and protects the baby B. It maintains the T of the baby C. It is the way the baby gets food and O2 D. It prevents Abs and viruses from passing to the baby E. It provides an exchange of nutrients and waste products between the mother and developing fetus

Answer: C and E (D is not a correct answer, because that is the downfall of the placenta is that it cannot prevent Abs and viruses from passing to the baby/ this is why HIV can be spread to the fetus!*)

A 12-month-old with Kawasaki disease received IV immunoglobulin (IVIG) 2 months ago. The child is in the clinic for follow-up and scheduled immunizations. Which vaccine should be delayed? Select all that apply. A. Haemophilus influenza type b (Hib) B. Hep B C. MMR D. Pneumococcal conjugate (PCV) E. Varicella

Answer: C and E B/c both of these are LIVE vaccines and IVIG administrative therapy may decrease the child's ability to produce the appropriate amount of antibodies to provide lifelong immunity. -Hib recommended between age 12-15 months. -Hep B final dose (#4) administered by age 6-18 months. - PCV final dose recommended between 12-15 months

The charge nurse is instructing a new grad RN on performing postmortem care. Which client situations might cause the RN to delay or not perform postmortem care? Select all that apply. A. Client died following a prolonged illness B. Client's family was not present when death occurred C. Client's religious background requires special ceremonial treatment of the body D. Death occurred in the ED following a suicide attempt E. Family requests a priest to perform last rites

Answer: C, D, and E

The nurse practicing in an out-patient clinic cares for a client recently diagnosed with hyperthyroidism. Which diet-related teaching should the nurse add to the client's plan of care? Select all that apply. A. Emphasize the importance of a low-carb diet B. Encourage the client to increase high-fiber foods in the diet C. Include meals and snacks high in protein content D. Teach avoidance of caffeine-containing liquids E. Teach the client about consumption of a high0cal diet of 4000-5000 cals/day

Answer: C, D, and E

What is the drug of choice to treat an SVT rhythm?

Adenosine

Measles

Airborne precautions

What laboratory value can help indicate if a blood clot was present?

An elevated D-dimer

Pulses paradoxus

An exaggerated fall in SBP over 10 mmHg during inspiration.

Define gender dyphoria

An incongruence between one's experienced and expressed gender and assigned gender of a duration of at least 6 months.

Infectious mononucleosis

An infection caused by the Epstein-Barr virus (EBV) that is characterized by fever, a sore throat, and enlarged lymph nodes. - "Kissing virus"

ID the type of ego defense mechanism from each example- 1. Someone not doing well academically who focuses on doing well in sports. 2. A husband with thoughts of infidelity who then accuses his wife of cheating. 3. A client with cancer who fears dying but behaves in an overly optimistic and fearless manner about his treatment and prognosis. 4. A client leaves a stressful family meeting and immediately begins to verbally abuse a roommate. Options: Compensation, Displacement, Projection, or Reaction Formation

Answer: 1. Compensation 2. Projection 3. Reaction formation 4. Displacement

The nurse is performing a CVC tubing change when the client suddenly begins gasping for air and writhing. Order the interventions by priority. All options must be used. - Place the client in Trendelenburg position on the left side - Stay with the client and provide assistance - Administer O2 PRN - Clamp the catheter tubing - Notify the HCP

Answer: 1. Clamp the catheter tubing 2. Place the client in Trendelenburg position on the left side 3. Administer O2 PRN 4. Notify the HCP 5. Stay with the client and provide assistance

A client requiring surgery is anxious about the possible need for a blood transfusion during or after the procedure. The nurse suggests to the client to take which action(s) to reduce the risk of possible transfusion complications? SELECT ALL THAT APPLY. 1) Ask a family member to donate blood ahead of time. 2) Give an autologous blood donation before the surgery. 3) Take iron supplements before surgery to boost hemoglobin levels. 4) Request that any donated blood be screened twice by the blood bank. 5) Take adequate amounts of vitamin C several days prior to the surgery date.

Answer: 1 and 2

What is the Schultz and Duncan Mechanism in the third stage of labor? (when the placenta is delivered)

"Shiny Schultz" and "Dirty Duncan" Schultz mechanism= shiny fetal side of the placenta Dunant mechanism= dull, rough maternal surface of placenta

Strabismus is?

"cross eyes"

The nurse is conducting a preschool assessment of a 4-year-old child. Which task, if not observed or reported by the parents as accomplished, will cause the nurse concern? Select all that apply. A. Draw a circle B. Jump rope with both feet C. Sit quietly for 30 min D. Use a spoon and fork E. Walk up and down stairs

.Answer: A, D, and E

The nurse is preparing to care for a dying client, and several family members are at the clients bedside. Which therapeutic techniques should the nurse use when communicating with the family? Select all that apply. A. Discourage reminicising B. Make the decisions for the family C. Encourage expression of feelings, concerns, and fears D. Explain everything that is happening to all family members E. Touch and hold the clients or family members hand if appropriate F. Be honest and let the client and family know they will not be abandoned by the nurse.

Answer: C, E, and F

A client with a mandibular fracture who has the upper and lower teeth wired together begins to choke on excessive oral secretions. What is the nurse's immediate action? A. Cut the wires B. Elevate the HOB C. Notify the HCP D. Suction the mouth and oropharynx

Answer: D

A nurse is assessing a newborn with an infection due to Candida albicans. Which assessment data support this diagnosis? A. Diffuse skin rash that resembles flea bites B. Small, white cysts on the hard palate C. Vesicles on the skin surrounding the lips D. White, adherent patches on the tongue and palate

Answer: D

The charge nurse on a medical-surgical unit is helping a student nurse formulate a care plan for a client with constipation. Which intervention in the care plan would cause the charge nurse to intervene? A. Allow the client to ambulate in the hall as tolerated B. Encourage the client to increase intake of nuts and seeds C. leave the client alone in the room when using the restroom D. Request coffee to be included with breakfast trays

Answer: D

The nurse reinforces the physical therapist's teaching regarding the use of a cane when caring for a client with osteoarthritis of the left knee. Which client statement indicates the need for further teaching? A. "I will hold the cane in my right hand" B. "I will not move my leg forward after moving the cane" C. "I will place the cane several inches in front of and to the side of my right foot" D. "My cane should equal the distance from my waist to the floor"

Answer: D

1 gram of weight is equal to how many mL?

1 gram of weight = 1 mL of fluid

What is the conversion from grams to mL when weighing a baby's diaper?

1 gram= 1 mL

Saturating a peripad within how many hours could indicate postpartum hemorrhage?

1-2 hours

Infant milestones: 1. When does he/she hold up head? 2. When does he/she roll over? 3. When does he/she sit up alone? 4. When does he/she crawl? 5. When does he/she stand and walk alone? 6. When does he/she start teething? 7. When are solid foods introduced? 8. At what time can eggs be given?

1. 2 months - holds up head 2. 4 months - rolls from abdomen to back 6 months- rolls over unassisted 3. 8 months - sits up alone 4. 10 months - crawls 5. 12 months - standing with support & walking neonate - palmer grasp reflex 6. Teething at 6 months 7. 5-6 months 8. 12 months (1 year old)

What are the clinical sxs of cold stress? 1. Neurological 2. Cardiovascular 3. Respiratory 4. GI 5. Musculoskeletal

1. Altered mental status (irritability or lethargy) 2. Bradycardia 3. Tachypnea early, followed by apnea and hypoxia 4. high gastric residuals, emesis (vomiting), hypoglycemia 5. hypotonia, weak suck and cry

Aldosterone secretion: What diseases are characterized by... 1. Increased aldosterone secretion 2. Decreased aldosterone secretion

1. Cushing's syndrome (causes hypoK+) - B/c aldosterone causes increased K+ excretion. 2. Addison's disease (causes a reduced K+ excretion= hyperkalemia)

What is chest tube output that should be reported to the HCP immediately? (2)

1. Drainage greater than 3 mL/kg/hr for 3 consecutive hours OR 2. Drainage greater than 5-10 mL/kg in 1 hour

What are the 3 types of solutions?

1. Isotonic: (a) 0.9% saline (NS) (b) LR (c) D5W (initially isotonic, then hypotonic shortly after administration) 2. Hypertonic: (a) D5 1/4 NS (b) D5 1/2 NS (c) D5NS (0.9%) (d) D5LR 3. Hypotonic: (a) D5W

Client instructions when using a Holter monitor (3)

1. Keep a diary of activities and any symptoms experienced while wearing the monitor. 2. Do NOT bathe or shower during the test period (24-48 hours). 3. Engage in normal activities to stimulate conditions that may produce sx's that the monitor can record.

What are lifestyle changes to reduce heartburn during pregnancy? (5)

1. Keeping the HOB elevated using pillows 2. Sitting upright after meals 3. Eating small, frequent meals 4. Avoiding tight-fitting clothing 5. Eliminating common dietary triggers (Ex: fried/fatty foods, caffeine, citrus, chocolate, spicy foods, tomatoes, carbonated drinks, peppermint)

1. What is the L/S ratio? 2. What is it used for?

1. Lecithin/sphingomyelin ratio 2. A test of fetal amniotic fluid to assess for fetal lung immaturity. Should be greater than 2:1.

Classify the type of insulin (action) 1. Glargine/ detemir 2. NPH 3. Insulin aspart/ lispro 4. Insulin reg

1. Long-acting insulin 2. Intermediate acting insulin 3. Rapid-acting insulin 4. Short acting insulin

What are the 4 phases of disaster management?

1. Mitigation - Prevent/reduce damage 2. Preparedness - Plan for rescue 3. Response - Disaster plan services into action 4. Recovery - Restore community stability

What are the 3 identifiers?

1. Name 2. DOB 3. MRN #

Blood transfusion: 1. What solution do you initiate the infusion with? 2. How long do you monitor the patient for a transfusion reaction?

1. Normal saline (prime the tubing with it first before spiking the blood transfusion bag. The patient will receive NS before the blood product.) 2. Monitor for reaction with the first 15 minutes following administration.

What are the 4 phases of seizure manifestations?

1. Prodromal phase: period with warning signs preceding the seizure/ before the aural phase. 2. Aural phase: period before the seizure that the client may experience visual or other sensory changes. - *Not all clients experience or can recognize a prodromal or aural phase before a seizure. 3. Ictal phase: period of ACTIVE seizure activity. 4. Postictal phase: client may experience confusion or also headache (HA) while recovering from the seizure.

What are the types of power? (7)

1. Reward= ability to provide incentives. 2. Coercive= ability to punish. 3. Referent= based on interaction. 4. Expert= based on having expert knowledge and skill level. 5. Legitimate= based on a position in society. 6. Personal= derived from a high degree of self-confidence. 7. Informational= when 1 person provides explanations why another should behave in a certain way.

What are the 4 types of wound drainage?

1. Serous (clear; watery) 2. Sanguineous (bright red bloody drainage; indicative of active bleeding) 3. Serosanguineous (pale red and watery; pinky) 4. Purulent (thick yellow, green, tan, or brown)

A client receiving a first dose of IV cefazolin has developed a diffuse rash, hypotension, and shortness of breath. Place the nurse's subsequent actions in the correct order. All options must be used. - Stop the infusion and call for help - Monitor VS for changes - Give IM Epinephrine and start IV NS 0.9% - Administer diphenhydramine IV - Assess airway and place client on O2

1. Stop the infusion and call for help. 2. Assess airway and place client on O2. 3. Give IM epinephrine and start IV NS 0.9%. 4. Administer diphenhydramine IV. 5. Monitor VS for changes.

What are the 4 signs of extravasation (infiltration of a drug into the tissue surrounding the vein)?

1. Swelling 2. Redness 3. Pain 4. Blanching

What are the different types of nursing delivery systems? 1. Functional Nursing 2. Team Nursing (Modular Nursing) 3. Relationship-based practice/ Primary Nursing 4. Client-focused care/ Total Nursing Care

1. Task-oriented - Team member focuses on delegated task rather than the total client= fragmented care 2. Led by RN team lead to determine work assignment. - "Modular Nursing" is similar but considers unit structure. Unit divided into modules= allows uses to care for clients geographically close by. 3. Nurse is actively involved in client care. - THINK: VA Home-Based Primary Care RNs - 1 RN responsible for managing and coordinating client's care in hospital and discharge. 4. RN assumes total responsibility for plan and delivering care (RN provides all necessary care).

What are some examples of advanced directives?

1. The living will 2. Durable power of attorney for health care / health care power of attorney 3. The Five Wishes® and directives like: - "Do Not Resuscitate" orders - Physician Orders for Life-Sustaining Treatment - Pregnancy - Organ and tissue donation

It is diagnosed A. fib IF (2)

1. There is NO P wave 2. Its rhythm is IRREGULAR

What are contraindications/ clients who should NOT receive tPA?

1. Thrombocytopenia (platelets less than 100,000) 2. HEMORRHAGIC STROKE 3. Uncontrolled HTN 4. Stroke or head trauma within the past 3 months 5. Major surgery within the past 14 days

Clinical manifestations of a spontaneous tension pneumothorax (4)

1. Tracheal deviation 2. Absent lung sounds 3. Severe abrupt hypoTN 4. Dyspnea

A parent of a 3 year old tells a clinic nurse that the child is rebelling constantly and having temper tantrums. Using Erikson's psychosocial development theory, which instructions should the nurse provide to the parent? Select all that apply. A. Set limits on the child's behavior. B. Ignore the child when this behavior occurs. C. Allow the behavior, because this is normal at this age period. D. Provide a simple explanation of why the behavior is unacceptable. E. Punish the child every time the child says "no" to change the behavior.

Answer: A and D Not E because this option would likely produce a negative response during this normal developmental pattern for a 3 year old.

The nurse is providing education to a client diagnosed with trichomoniasis vaginal infection who has been prescribed a onetime dose of oral metronidazole. Which of the following statements by the nurse are appropriate? Select all that apply. A. "Abstain from sexual intercourse until sx's are cleared" B. "Avoid drinking ETOH for at least 3 days after taking the last dose" C. "Inform you sexual partners that they need to be treated" D. "Metronidazole may temporarily turn your urine a dark, brownish color" E. "Vaginal douching after intercourse may prevent recurrence of infection"

Answer: A, B, C, and D

A client with a hip fracture is placed in Buck traction. Which activities are appropriate for the nurse to include in the client's plan of care? Select all that apply. A. Assess for skin breakdown of the limb in traction B. Ensure adequate pain relief C. Keep the limb in a neutral position D. Perform frequent neuromuscular checks on the limb in traction E. Reposition the client and use a wedge pillow

Answer: A, B, C, and D Not E= b/c side-to-side repositioning can cause injury Note: Buck traction is used to immobilize HIP FRACTURES and reduce pain and spams util the client can undergo surgical repair of the fracture. Limb needs to remain in a straight and neutral position at all times and the client is in a SUPINE position OR semi-Fowler's position (maximum 20-30 degrees).

The nurse is preparing to administer meds using a clients NG tube. Which actions should the nurse take before administering the med? Select all that apply. A. Check residual volume B. Aspirate stomach contents C. Turn off suction to the NG tube D. Remove the tube and place it in the other nostril E. Test the stomach contents for a pH indicating acidity

Answer: A, B, C, and E

The nurse reinforces teaching about self-management strategies for a client with urge incontinence. Which of the following statements indicate that teaching has been effective? Select all that apply. A. "I am going to join a walking program to lose excess weight" B. "I may have dry mouth as a side effect from the oxybutynin" C. "I really need caffeine to get myself going in the morning" D. "I should perform Kegel exercises several times daily" E. "I will void q 2 hr until I am having fewer accidents"

Answer: A, B, D, and E

The nurse is preparing to defibrillate a client who suddenly went into ventricular fibrillation. Which steps are essential prior to delivering a shock? Select all that apply. A. Apply defibrillator pads B. Call out and look around to ensure that everyone is "all clear" C. Continue chest compressions until ready to deliver shock D. Ensure adequate IV sedation has been given E. Ensure that the synchronization button is turned on

Answer: A, B, and C Not E= b/c do not want that button on in V fib

The clinic nurse prepares to perform a focused assessment on a client who is complaining of symptoms of a could, cough, and lung congestion. Which should the nurse include for this type of assessment? Select all that apply. A. Auscultating lung sounds B. Obtaining T C. Assessing peripheral pulses D. Obtaining info about the clients respirations E. Performing a musculoskeletal and neuro exam F. Ask the client about family history of any illness or disease

Answer: A, B, and D

A parent calls the nurse telehealth triage line with concerns about an allergic reaction to something a child ate. Which symptoms should the nurse instruct the parent to assess for to determine if the child is having an anaphylactic reaction? Select all that apply. A. Dyspnea B. Fever C. Lightheadedness D. Skin rash (hives) E. Wheezing

Answer: A, C, D, and E

A nurse is caring for an intubated client receiving a continuous sedative infusion. Which interventions by the nurse reflect correct understanding of preventing ventilator-acquired pneumonia? Select all that apply. A. Elevating the HOB to 30-45 degrees B. Performing hours in-line ET suctioning C. Practicing strict hand hygiene D. Providing frequent oral care with chlorhexidine E. Scheduling daily sedation vacations

Answer: A, C, D, and E Not B= b/c ET suctioning should only be performed when clinically indicated. B/c frequent suctioning increases risk for tracheal and bronchial trauma, bleeding, and hypoxia

A teenage client with sickle cell dz reports having a vaso-occlusive crisis (pain crisis). family members say that the client is just "drug seeking." Which expected lab findings would help confirm the presence of a sickle cell crisis? A. Elevated bilirubin B. Elevated eosinophils C. Elevated reticulocyte count D. Hgb less than 10 E. K+ less than 3.5

Answer: A, C, and D

The charge nurse assists a student nurse preparing to apply knee-length compression stockings onto a client with chronic venous insufficiency. Which actions by the student nurse would cause the charge nurse to intervene? Select all that apply. A. Instructs client that stockings will be worn only at night B. Measure circumference of both calves at the widest point C. Rolls down any excess length at the top of the stocking D. Selects a size larger to avoid friction against a leg laceration E. Smoothes out any wrinkles or creases in the stocking

Answer: A, C, and D

The nurse is caring for a client who had a stroke two weeks ago and has moderate receptive aphasia. Which interventions should the nurse include in the plan of care to help the client follow simple commands regarding activities of daily living (ADL)? Select all that apply. A. Ask simple questions that require Y/N answers B. If the client becomes frustrated, seek a different care provider to complete ADL C. Remain calm and allow the client time to understand each instruction D. Show the client pics of ADL (Ex= shower, toilet, and toothbrush) or use gestures E. Spak slowly but loudly while looking directly at the client

Answer: A, C, and D

The nurse plans care for a pediatric client who has just undergone a cleft palate repair. Which of the following interventions should the nurse include in the plan of care? Select all that apply. A. Assist and encourage caregivers to hold and comfort the child B. Offer pacifier in between feedings to promote the childs comfort C. Position the child supine with an elevated HOB after feedings D. Remove elbow restraints per policy for skin and circulatory assessment E. Use tongue blade and penlight to assess surgical site q 4 hours

Answer: A, C, and D

A client with advanced osteoarthritis is admitted for right total knee arthroplasty. Which characteristic manifestations does the nurse expect to assess in this client? Select all that apply. A. Crepitus with joint movement B. Low-grade fever C. Morning stiffness lasting 10-15 min D. Pain exacerbated by wt-bearing activities E. + serum rheumatoid factor

Answer: A, C, and D Not B= b/c a low-grade fever develops as part of systemic inflammation. Osteoarthritis is typically a noninflammatory, nonsystematic d/o (inflammation is limited to affected joints).

A client sustained a concussion after falling off a ladder. What are essential instructions for the nurse to provide when the client is discharged from the hospital? Select all that apply. A. Client should abstain from ETOH B. Client should remain awake all night C. Client should return if having difficulty walking D. Responsible adult should be taught neuro exam E. Responsible adult should stay with the client

Answer: A, C, and E Not B b/c not necessary to cause sleep deprivation by keeping the client awake. Not D b/c this should be done by a clinician.

A client is suspected of having Graves' disease (hyperthyroidism). Which signs and/or symptoms are expected to be present in this client? Select all that apply. A. Anxiety B. Bradycardia C. Dry skin D. Heart palpitations E. Protrusion of the eyeballs F. Weight gain

Answer: A, D, and E

A client with suspected moderate to large pericardial effusion (fluid starts pooling into the pericardial space) is admitted for monitoring. The nurse performs a head-to-toe assessment. Which of these findings indicate likely cardiac tamponade and require immediate intervention? Select all that apply. A. BP 90/70 B. Bounding peripheral pulses C. Decreased breath sounds on left side D. Distant heart tones E. JVD (Can you list other sx's?) [What is Beck's Triad?]

Answer: A, D, and E (Other sx's include): - Pulsus paradoxus (Sbp drop greater than 10 mmHg with INSPIRATION of the lungs!) - Dyspnea/ tachypnea - Tachycardia [1. JVD, 2. HypoTN, and 3. Distant heart sounds]

The nurse is caring for a client admitted for a seizure disorder. The nurse witnesses the client having a tonic-clonic seizure with increasing salivation. Which actions should the nurse take? Select all that apply. A. Call for help B. Hold down the clients arms C. Insert tongue depressor to move tongue D. Prepare for suctioning E. Turn the client on the side

Answer: A, D, and E Not B b/c restraining client could cause an injury!

The nurse is caring for a client on the medical-surgical unit who is positive for delirium according to the Confusion Assessment Method (CAM) assessment tool. Which of the following assessment findings are likely contributing to the client's delirium? Select all that apply. A. 3 doses of hydromorphne 2mg IVP in the past 12 hours B. Serum Na+ of 125 C. SpO2 82% on room air D. T 101.5 F E. Urine culture is + for gram + cocci in chains

Answer: All of the above

A fever is diagnosed with a T of...?

100.4 F and above

A self-employed auto mechanic is diagnosed with CO poisoning. Admission VS are BP 90/42, HR 84, RR 24, and SpO2 94% on room air. What is the nurse's priority action? A. Admin 5 mg inhaled albuterol nebulizer tx to decrease inflammatory bronchoconstriction B. Admin 100% O2 via non-rebreather at 15 L/min C. Admin methylprednisolone to decrease lung inflammation from toxic inhalant D. Titrate O2 to maintain pulse ox above 95%

Answer: B (B is right b/c administering O2 quickly will help in reversing this displacement of O2) Not A or C b/c lung inflammation is not the issue with CO poisoning. Not D b/c pulse ox will NOT DETECT CO poisoning. It will indicate that the SpO2 is 100%.

When the nurse provides education about starting risperidone, which statement by the client's caregiver indicates a need for further teaching? A. "I will call the clinic if the client has a fever or muscle stiffness" B. "I will remind the client to move slowly and not stand up too quickly" C. "I won't worry if the client sleeps more often when taking this med" D. "It is normal for the client to become shaky and restless when agitated"

Answer: D Not A= b/c these signs may indicate neuroleptic malignant syndrome and can be potentially fatal requiring emergent intervention

What is a normal Central Venous Pressure (CVP)?

2-8 mmHg *An elevated CVP can indicate R-sided HF or fluid volume overload.

How many lbs are in 1 kg?

2.2 lbs = 1 kg

A nurse is caring for a client on a mechanical ventilator. The ventilator is sounding an alarm and displaying an alert about low tidal volumes. The nurse has checked all connections and the endotracheal tube, but the alarm persists and the client's oxygen saturation is dropping. What should the nurse do next? A. Call the RT to the bedside to troubleshoot B. Elevate the HOB and apply a nonrebreather mask C. Increase the O2 delivery on the ventilatory to 100% D. Manually vent with a resuscitation bag device attached to the ETT

Answer: D Not B= b/c since the client is intubated, air cannot pass from the nares and oropharynx into the lungs, and ventilation can be achieved only via the ETT.

The nurse cares for a group of clients on a medical surgical floor. The client with which condition is at highest risk for developing syndrome of inappropriate antidiuretic hormone (SIADH)? A. Carpal tunnel syndrome B. DM C. Sciatica D. Small cell lung cancer

Answer: D Note: Sciatica is numbness, tingling, or pain caused by an irritation of the sciatic nerve.

The nurse caring for a client with a pneumothorax and who has had a chest tube inserted notes continuous gentle bubbling in the water seal chamber. What action is most appropriate? A. Do nothing, because this is an expected finding B. Check for an air leak, because the bubbling should be intermittent C. Increase the suction pressure so that the bubbling becomes vigorous D. Clamp the chest tube and notify HCP immediately

Answer: B Fluctuation with insertion and expiration, not continuous bubbling, should be noted in the water seal chamber. Intermittent bubbling may be noted with patient has a known pneumothorax.

Which one is a K+ sparing diuretic? A. Phenytoin B. Spirolactone C. Furosemide

Answer: B Furosemide is a K+ depleting loop diuretic. HypoK+ can lead to heart palpitations and dysrhythmias.

The nurse is caring for a newborn with patent ductus arterioles. Which assessment finding should the nurse expect? A. Harsh systolic murmur B. Loud machine-like murmur C. Soft diastolic murmur D. Systolic ejection murmur

Answer: B Many newborns are asymptomatic except for a loud, machine-like systolic and diastolic murmur.

A client undergoes transurethral resection of the prostate for benign prostatic hyperplasia. The client has a 3-way Foley catheter with continuous bladder irrigation. Which assessment is the best indication that the bladder irrigation flow rate is productive? A. BP 120/80 with HR 80 B. Client has no bladder spasms C. Irrigation input 3000mL, Foley output 3000mL D. Output urine is light pink in color

Answer: D RN should adjust irrigation rate with these normal findings so that the urine remains light pink w/o clots. Note: For continuous bladder irrigation, the total foley output should be more than the CBI input (b/c output is CBI fluid PLUS the normal renal output). - An obstruction is indicated if the CBI input is equal or greater than the Foley output

The pediatric nurse plans a home visit for a 2-year-old who will soon be discharged with home health care. Which condition presents the most concern as a safety hazard in the child's home environment? A. Family lives in a rural area B. House is heated by a wood-burning stove C. House was built in 1983 D. Parents are unemployed with limited financial resources

Answer: B Not C= b/c it is houses built before 1978 that have a high probability of containing lead-based paint. Active renovations can significantly increase the amount of lead released into the home environment, leading to lead poisoning (neuro and motor impairment). Living in a house built in 1983 is not associated with increased risk of lead exposure.

Upon arrival in the post-anesthesia care unit, the nurse performs the initial assessment of a client who had surgery under general anesthesia. Which assessment finding prompts the nurse to notify the health care provider immediately? A. Difficult to arouse B. Muscle stiffness C. Pinpoint pupils D. T 94 F (34.4 C)

Answer: B Not C= b/c small pupil size are associated with drugs used to induce general anesthesia, sedating drugs, and opioid drugs to control pain (this is an expected finding in the PACU)*

The nurse helps a client with end-stage renal disease and a serum potassium level of 5.2 mEq/L (5.2 mmol/L) to plan menu choices. Which items would be best to include in the meal plan? A. Black beans and rice, sliced tomatoes, half a cantaloupe B. Grilled chicken sandwich on white bread, applesauce C. Hamburger patty on whole wheat bun, carrot sticks, chocolate pudding D. Poached salmon, greens peas, baked potato, strawberries

Answer: B Not D= b/c potatoes and strawberries are both foods highest in K+ Yes B= b/c apple products and chicken are one of the foods lowest in K+

The nurse is performing discharge teaching on nutritional therapy for a client with chronic kidney disease. Which statement indicates that further teaching is needed? A. "B/c I have chronic kidney dz, I should avoid canned soups and cold-cut sandwiches" B. "I can use a salt substitute b/c I am required to restrict both Na+ and K+ in my diet" C. "I must avoid eating raw carrots and tomatoes on my salads b/c I take hemodialysis tx's" D. "The popsicles I eat should be counted in my daily fluid intake b/c they become liquid at room T"

Answer: B Yes B= b/c CKD clients should AVOID salt substitutes, which typically contain KCl and may contribute to hyperK+ Not C= b/c this is true, raw carrots, tomatoes, and orange juice are high in K+ and should be avoided in clients with CKD.

Urine output normally should be how many mL/hr AT LEAST?

30 mL/hr

The clinic nurse is instructing a client who is newly prescribed transdermal scopolamine to prevent motion sickness during an upcoming vacation on a cruise ship. Which of the following statements made by the nurse are appropriate? Select all that apply. A. "Apply the patch when the ship starts moving and not before" B. "Dispose of the patch out of reach of children and pets" C. "Make sure to remove the old patch before applying a new one" D. "Place the patch on a hairless, clean, dry area behind the ear" E. "Wash your hands with soap and water after handling the patch"

Answer: B, C, D, and E

Which situations require that the registered nurse (RN) report to an appropriate authority? Select all that apply. A. Client has a row of 3-inch circles down the back from "cupping" B. Client is diagnosed with gonorrhea and requests not to report under HIPAA C. RN thinks a teenage client's signs are from abuse, but the HCP does not D. RN thinks an elderly client's signs are from abuse but the client denies this E. Syphilis is diagnosed in an 11 y/o who denies sexual activity

Answer: B, C, D, and E

How long should you auscultate bowel sounds before you can determine that they are absent?

5 minutes

A tepid bath indicates?

A "lukewarm" bath temperature. *Want to use this when atopic dermatitis (eczema) is diagnosed.

What size syringe should be used to administer medications into an Central Venous Catheter (CVC) line?

A 10mL syringe or bigger when flushing CVCs and implanted ports.

When taking MAOIs like Phenelzine, what foods should be restricted/avoided to reduce the risk for hypertensive crisis? Select all that apply. A. A biscuit B. Broad beans C. Red wine D. Chocolate E. Avocados

Answer: B, C, D, and E These foods are high in tyramine, which need to be avoided when taking MAOIs.

The nurse educator is preparing to conduct a teaching session for the nursing staff regarding the theories of growth and development and plans to discuss Kohlberg's theory of moral development. What info should the nurse include in the session? Select all that apply. A. Individuals move through all 6 stages in a sequential fashion. B. Moral development progresses in relationship to cognitive development. C. A person's ability to make moral judgements develops over a period of time. D. The theory provides a framework for understanding how individuals determine a moral code to guide their behavior. E. In stage 1, children are expected to reason as mature members of society. F. In stage 2, the child conforms to rules to obtain rewards or have favors returned.

Answer: B, C, D, and F

A large-scale community disaster occurs and clients must share hospital rooms due to the rapid influx of new victims. Which room assignments are appropriate in this situation? Select all that apply. A. 2 clients on contact isolation, 1 with vance-resistant enterococci infection and another with MRSA B. 2 clients with C diff infection, one in the stool and other in a wound C. A client in nickel cell dz crisis and a client with strep pneumonia D. A client who had an abdominal surgery today and a client with universal precautions E. A young client in Buck's traction with an elderly client with Parkinson's dz

Answer: B, D, and E

A nurse in the emergency department is caring for a homeless client just brought in with frostbite to the fingers and toes. The client is experiencing numbness, and assessment shows mottled skin. Which interventions should be included in the client's plan of care? Select all that apply. A. Apply occlusive dressings after rewarming B. Elevate affected extremities after rewarming C. Massage the areas to increase circulation D. Provide adequate analgesia E. Provide continuous warm water soaks

Answer: B, D, and E

The intensive care nurse is caring for a client who has just been extubated. Which interventions are appropriate at this time? Select all that apply. A. Admin prescribed oral narcotics for throat pain B. Admin warmed, humidified O2 via facemask C. Give the client ice chips to moisten the mouth D. Provide mouth care with oral sponges E. Start the client on incentive spirometer

Answer: B, D, and E Not A or C= b/c clients are kept NPO after extubation to prevent aspiration. They may have either a bedside swallow screen or a more formal swallow eval by a speech therapist prior to swallowing any food, drink, or meds.

A nurse is performing cardiopulmonary resuscitation (CPR) on an adult at a swimming pool. A bystander brings the automated external defibrillator (AED). The nurse notes that the victim is wet, lying in a small pool of water, and wearing a transdermal medication patch on the upper right chest. What is the most appropriate action at this time? A. Do not use the AED and continue CPR until paramedics arrive B. Move the client away from the pool of water before applying AED pads C. Remove the transdermal patch and wipe the chest dry before using AED D. Wipe the chest ry and apply the AED pads over the transdermal patch

Answer: C

A student nurse is accompanying the charge nurse when conducting daily rounds. Which personal protective measure by the charge nurse does the student nurse question? A. Dons a mask with eye shield before irrigating a draining wound for a client on standard precautions B. Places a "soap and water only" sign on the door of a client with C diff C. Wears 2 pairs of gloves when emptying the urinary catheter collection bag of a client with HIV D. Wears a N95 respirator before entering the room of a client with active varicella-zoster

Answer: C

An older adult client takes multiple prescription medications plus several over-the-counter medications. Which intervention by the clinic nurse is most important in reducing the risk for drug interactions? A. Assist client with making a list of meds, doses, and times to be taken B. Encourage client to obtain all prescription meds from the same pharmacy C. Have client bring all meds taken regularly or occasionally to each appointment D. Instruct client to use a pill organizer to separate pills by day and time

Answer: C

The nurse creates a plan of care for a client with DVT. Which client position or activity in the plan should be included? A. Out of bed activities as desired B. Bed rest with the affected extremity kept flat C. Bed rest with elevation of the affected extremity D. Deb rest with the affected extremity in a dependent position

Answer: C

The nurse has conducted pre-op teaching for a client scheduled for surgery in 1 week. The client has a history of arthritis and has been taking aspirin (acetylsalicylic acid). The nurse determines that the client needs additional teaching if the client makes which statement? A. "ASA can cause bleeding after surgery" B. "ASA can cause my ability to clot blood to be abnormal" C. "I need to continue to take ASA until the day of the surgery" D. "I need to check with my HCP about the need to stop the ASA before the scheduled surgery"

Answer: C

The nurse is assigned to care for a client who had a thyroidectomy 24 hours ago. On initial assessment, which finding requires the most immediate action by the nurse? A. Ca2+ 8.8 B. HR 110 C. Laryngeal stridor D. Pain 8/10

Answer: C

The nurse is caring for a client who has been receiving mech vent for 4 days. During rounds, the HCP questions the development of ventilator-associated pneumonia. Which manifestations does the nurse assess as the best indicator of VAP? A. Blood-tinged sputum B. + blood cultures C. +, purulent sputum culture D. Rhonchi and crackles

Answer: C

The nurse is caring for a client with a fractured femur. Which nursing intervention is most effective in helping reduce the risk for fat emboli? A. Admin prophylactic enoxaprin as prescribed B. Frequent use of incentive spirometry C. Minimizing movement of the fractured extremity D. Use of an intermittent pneumatic compression device

Answer: C

The nurse is counseling a pregnant client who is HIV positive. Which information is appropriate to discuss? A. Infant should be exclusively breastfed for 6 months to receive maternal Abs B. Infant will not require treatment for HIV after birth C. Prescribed antiretroviral therapy should be continued during pregnancy D. Tetanus-diphtheria-pertussis vaccine should be avoided until after birth

Answer: C

The nurse is performing an Apgar assessment on a newborn client at 1 minute of life. The newborn is completely blue, has a HR 110/min, and is emitting a weak cry. Active movement and flexion of extremities are noted and the newborn grimaces when nares are suctioned. Which Apgar score should the nurse assign this newborn? A. Apgar score of 4 B. Apgar score of 5 C. Apgar score of 6 D. Apgar score of 8

Answer: C

The nursing instructor asks the student to describe fetal circulation, specifically the ductus venosus. Which statement by the student indicates an understand of the ductus venosus? A. "It connects the pulmonary artery to the aorta" B. "It is an opening between the right and left atria" C. "It connects the umbilical vein to the inferior vena cava" D. "It connects the umbilical artery to the inferior vena cava"

Answer: C

When an unlicensed assistive personnel (UAP) assists a client with a chest tube back to bed from the bedside commode, the plastic chest drainage unit accidently falls over and cracks. The UAP immediately reports this incident to the nurse. What is the nurse's immediate action? A. Clamp the tube close to the client's chest until a new chest drainage unit is set up B. Notify HCP C. Place the vital end of the chest tube into a bottle of sterile NS D. Position client on the left side

Answer: C

The nurse is caring for a client with sepsis and acute respiratory failure who was intubated and prescribed mechanical ventilation 3 days ago. The nurse assesses for which adverse effect associated with the administration of positive pressure ventilation (PPV)? A. Dehydration B. HypoK+ C. HypoTN D. Increased CO

Answer: C + pressure applied to the lungs compresses the thoracic vessels and increases intrathoracic pressure during inspiration. This leads to reduced venous return, ventricular preload, and CO= results in hypoTN

Disulfiram (Antabuse)

A form of aversion therapy that promotes abstinence from alcohol.

Huntington's disease

A human genetic disease caused by a dominant allele; characterized by uncontrollable body movements and degeneration of the nervous system; usually fatal 10 to 20 years after the onset of symptoms.

What is fat embolism syndrome?

A life-threatening condition that has no specific tx. - Happens when a long bone is fractured, pressure within the bone marrow leads to the release of fat globules into the bloodstream. These fat globules combine with platelets to form a fat embolus. - To prevent this from happening, early STABILIZATION of the injury and surgery needs to occur ASAP to repair the long bone fracture to reduce further injury to the soft tissue

Antisocial personality disorder

A personality disorder in which the person (usually a man) exhibits a lack of conscience for wrongdoing, even toward friends and family members. May be aggressive and ruthless or a clever con artist. - Want to set FIRM LIMITS

Pilot balloon (its use on a double-lumen tracheostomy tube)

A pilot balloon is attached to the outside of the tube that helps in indicating the presence or absence of air in the cuff that should be inflated inside the trachea.

Catatonia

A state of immobility and unresponsiveness lasting for long periods of time. Commonly seen in clients with schizophrenia.

Aortic dissection

A tear in the inner lining of the aorta that allows blood to surge between the layers of the arterial wall= separation and weakening of the aortic wall.

What is refractory hypoxemia?

Hypoxemia unresponsive to increasing concentrations of O2. - This is related to the noncardiogenic pulmonary edema that occurs in the lungs from increased flow of fluid in the interstitial space. This fluid then proceeds to lead to alveolar edema as this fluid crosses the blood-gas barrier. - This is BAD, b/c the alveoli are flooded so no O2 can cross to oxygenate the body (gas exchange canNOT occur!)

Define battery

INTENTIONAL touching without others consent.

Encopresis

Involuntary defecation not attributable to physical defects or illness. A child over 4 y/o shits their pants.

Somnolence

Is a state of strong desire for sleep (sleepiness or drowsiness)

What is the main difference between where to place a lidocaine or capsicum patch versus a fentanyl patch?

Lidocaine and capsicum patches need to be placed locally to the painful site. Fentanyl patch is absorbed systemically therefore does not need to be placed at the painful site directly.

Ataxia

Loss of full control of bodily movements.

Diabetic ketoacidosis is respiratory or metabolic?

Metabolic acidosis *Note: Ketoacidosis is also associated with alcoholism and starvation)

The nurse understands that persistent vomiting place a patient at risk for developing which acid-base disorder?

Metabolic alkalosis

What are the levels in Maslow's Hierarchy of Needs?

Most basic... physiological needs--> safety-->love and belonging-->self-esteem--> self-actualization (Basic needs are met before moving to other needs in the hierarchy.)

What is the ONLY fluid that can be given with a blood transfusion?

Normal saline ONLY B/c RBCs are NOT COMPATIBLE with D5W.

A client with parenteral nutrition infusing has disconnected the tubing from the CVC. The nurse assesses the client and suspects an air embolism. The nurse should IMMEDIATELY place the client in which position?

On the left side, with the head lower than the feet (left side-lying Trendenburg position) Used to minimize the effect of the air traveling as a bolus to the lungs by trapping it in the right side of the heart.

What value is considered excessive drainage from a chest tube?

Over 100 mL/hr Note: Excessive drainage of frank blood is indicative of hemorrhage and must be managed immediately.

How long after opening a bottle of normal sterile saline solution is it still viable to use per agency policy?

Recapped bottles of solution are to be used within 24 HOURS of opening.

What is the "ROME" acronym when interpreting acid-base balance scenarios?

Respiratory OPPOSITE pH low; pCO2 high= resp acidosis pH high; pCO2 low= resp alkalosis Metabolic EQUAL pH low; HCO3 low= met acidosis pH high; HCO3 high= met alkalosis

What is Nagele's rule for estimating the date of delivery/ date of birth?

Take first date of last NORMAL menstrual period 1. Subtract 3 months 2. Add 7 days 3. Add 1 year

When the rupture of membranes occur, what will the nurse assess/document? (think TACO)

This is a rupture of the amniotic sac. "TACO" stands for... T= time A= amount C= color O= odor

Why would a 1-day old girl have a small amount of bloody mucus in her diaper?

This is due to mammary gland enlargement. AKA pseudo menstruation that is a benign transient finding commonly seen in newborns.

What are the roles of ethics committees?

To develop and establish policies and procedures to FACILITATE the prevention and resolution of dilemmas. *Important: these committees do not solve the problem, but help to do so.

Magnesium is given for what cardiac arrhythmia?

Torsades des pointes

What do clients attempting vaginal birth after cesarean (VBAC) have a slightly increased risk for?

Uterine rupture causing cessation of contractions, hypovolemic shock, and maternal tachycardia from the hemorrhage.

What is a meconium ileus?

When a newborns first stool (meconium) gets too thick and sticky (can be influenced by cystic fibrosis) and the stool gets STUCK in the INTESTINES. - This is a surgical emergency

What is magical thinking?

When one believes their thoughts may influence events. Conclusions based on desires of how things should work, rather than how they actually do. Ex: A four-year-old child, might believe that after wishing for a pony, one will appear at his or her house

What are high fiber foods?

Whole grains and bran, dried peas and beans, nuts, seeds, legumes, and fresh fruit and vegetables.

When is the expected time that the newborn would pass meconium/ first stool?

Within 24-48 hours

Do you lose or retain K+ when GI loses occur? (aka diarrhea/ vomiting)

You LOSE K+ with diarrhea and vomiting.

preeclampsia

abnormal condition associated with pregnancy, marked by high blood pressure, proteinuria, edema, and headache

What is fetal station?

relation of the presenting part of the fetus to the maternal ischial spines, measures the degree of descent of the fetus. Presenting part in line with the ischial spine= 0 Presenting part 2 cm above the ischial spine = -2 Presenting part 1 cm below the ischial spine =+1

What are the 5 Rights of Delegation?

right task right circumstance right person right direction/communication right supervision/evaluation

What is tissue plasminogen activator (tPA) do in cases of an ischemic stroke?

tPA dissolves clots and restores perfusion in clients with ischemic stroke. It is administered within a 3-4.5 hour window from the onset of sx's for full effectiveness. - Nurse must assess for contraindications to tPA due to the risk for hemorrhage.

What is the Romberg test?

the patient should be standing feet together and eyes closed, with minimal swaying. test neurological issues as well as ear imbalances.

A change in level of consciousness and seizures can occur with a low Na+ level of below what?

Below 120 mEq/L

NSAIDS like Ibuprofen, Aspirin, Ketorolac, Naproxen AND beta blockers have the potential to cause what in asthma patients?

Bronchospasm

Administering a rectal suppository in children less than 3 y/o versus a client over 3 y/o- What is the difference in... A. Position B. What finger to use during administration

Children less than 3 y/o- A. Place supine with knees and feet raised B. Fifth finger ("pinky") with suppository guided against the rectal wall Client over 3 y/o- A. Place side-lying with knees flexed B. Index finger

Difference between a cystocele versus rectocele?

Cystocele: vaginal wall and bladder prolapse into anterior orifice of vagina. Rectocele: prolapse of rectum into vaginal canal.

1. Infant age 2. Newborn age 3. School age 4. Preschool age 5. Adolescent age 6. Toddler age

1. 1 month to 1 year 2. Birth to 1 month 3. 6-12 years 4. 3-6 years 5. 13-18 years 6. 1-3 years

What are the three ways to check NG tube placement?

1. Aspiration 2. Inject air with stethoscope 3. Chest X-ray

1. VS include 2. What are the normal ranges?

1. Blood pressure, Pulse, Respirations, Temperature, and Pulse oxygen saturation (*Also pain) 2. BP under 120/ under 80 mmHg Pulse 60-100 bpm Respirations 10-20 breaths/min Temperature 36.4-37.5 C (97.5-99.5 F) Pulse oxygen saturation 95-100%

Crepitus- 1. What does it feel like? 2. What causes it?

1. Feels like crackling, crinkly or grating under the skin, around the lungs or in the joints. 2. Air bubbles popping inside the joint.

s/s of depression in adolescents

1. Hypersomnolence or insomnia (napping during daily activities). 2. Low self-esteem/ withdrawal from previously enjoyable activities. 3. Outbursts of angry, aggressive or delinquent behavior. 4. Weight gain or loss (increased food intake or lack of interest in eating).

Organs such as the heart, lungs, and liver can ONLY be obtained IF: (2)

1. Patient is on mechanical ventilation 2. Brain dead *Other organs/tissues can be removed for organ donation usually done by HCP or specially trained nurse.

What to do if a chest tube is pulled out of the chest accidentally? (4 steps)

1. Pinch the skin opening together. 2. Apply an occlusive sterile dressing. 3. Cover the dressing with overlapping pieces of 2-inch tape. 4. Call the HCP immediately.

Place the nurse's actions while awaiting the arrival of the rapid response team in priority order. - Place client in high Fowlers position - Notify HCP - Assess lung sounds - Admin 100% O2 nonrebreather mask - Perform orophargeal suctioning

1. Place client in high Fowlers position 2. Perform orophargeal suctioning 3. Admin 100% O2 nonrebreather mask 4. Assess lung sounds 5. Notify HCP

How to examine ear with an otoscope for an... 1. Adult 2. Children 3 years or younger

1. Pull pinna up and back 2. Pull pinna down and back

What is normal urine specific gravity?

1.005-1.030 (This is a measure of urine CONCENTRATION) - The higher the # (above 1.030), the more concentrated the urine= dehydration

A client with ascites had 5400 mL of fluid removed during paracentesis. The health care provider prescribes 8 g of albumin IV per 1000 mL of fluid removed. If the albumin is supplied as 25 g in 100-mL bottles, how many mL will the nurse administer? Record your answer using one decimal place.

172.8 mL

histrionic personality disorder

A personality disorder characterized by excessive emotionality and preoccupation with being the CENTER OF ATTENTION; emotional shallowness; overly dramatic behavior.

A. What is a normal ejection fraction? B. What is a normal LDL value (bad lipid)?

A. 55-70% B. Less than 100 mg/dL

Cellulitis- A. What is it? B. What is it characterized by? C. How should the affected extremity be placed when caring for these clients? D. What form of precautions are implemented?

A. Inflammation of the subcutaneous tissues that is typically caused by a bacterial infection resulting from an insect bite, cut, abrasion, or an open wound. B. 1. Redness 2. Edema 3. Pain 4. Fever C. Affected extremity is elevated when the client is sitting or lying down to promote lymphatic drainage. D. Standard precautions, BUT a gown and gloves are worn when contact with body fluids (ex= urine or stool) or potentially infectious drainage is expected, such as during bathing.

Hirschsprung disease- A. What is it? B. What are the clinical features? (4) C. What is a fatal complication of Hirschsprungs dz? D. What are the clinical sxs if this complication was present? (4)

A. Involves missing nerve cells in the muscles of the colon. Making it hard to pass stool. B. 1. Bilious vomiting (green bile from the liver) 2. Abdominal distension (from enlarged/megacolon) 3. Failure to pass meconium 4. Failure of internal anal sphincter relaxation C. Hirschsprung enterocolitis (inflammation of the colon, which can lead to sepsis and death) D. 1. Fever 2. Lethargy 3. Explosive, foul-smelling diarrhea 4. Rapidly worsening abdominal distention

A. What is a sickle cell crisis? B. Sickle cell crisis would have what laboratory findings? C. Tx for this includes...

A. Pain that can begin suddenly and last several hours to days caused by when sickled RBCs block small bl vessels that carry bl to your bones. B. 1. Elevated bilirubin due to Hgb breakdown from the excessive hemolysis 2. Elevated reticulocyte count from the bone marrow's activity in response to the anemia (due to RBCs sickling with destruction) 3. Hgb less than 10 g/dL (acute anemia usually occurs during sickle cell crisis and is due to the increased hemolysis of the RBCs; transfusions may be required) C. 1. Pain management with opioids 2. IV fluids for hydration (b/c avoiding dehydration is important with those with sickle cell anemia) 3. Bed rest to decrease energy expenditure and O2 demand

BMI ranges- A. Underweight B. Normal weight C. Overweight D. Obese

A. less than 18.5 B. 18.5-24.9 C. 25-29.9 D. 30 or more

What assessment findings are consistent with infiltration? A. Pain and erythema B. Pallor and coolness C. Numbness and pain D. Edema and blanched skin E. Formation of a red streak and purulent drainage

Answer: B, C and D

A client is admitted to the intensive care unit with diagnoses of a brain tumor complicated by transient diabetes insipidus. Which client data related to this complication should the nurse expect? Select all that apply. A. Dark amber urine with sediment B. High serum osmolality C. Low urine specific gravity D. Recent weight gain E. Reports of excessive thirst

Answer: B, C, and E Yes B= b/c of the HYPERNa+ that occurs with DI Yes C= b/c the urine is very dilute from the lack of ADH not holding onto water leading to urine containing A LOT of water (urine specific gravity less than 1.003/ 1.005) Yes E= b/c polydipsia occurs with DI (along with polyuria)* Not D= b/c the polydipsia and polyuria can lead to dehydration and weight loss from huge drop in water weight.

The emergency department nurse cares for 5 clients. Which of the clients below are at risk for developing metabolic acidosis? Select all that apply. A. 25 y/o with claustrophobia who was stuck in an elevator for 2 hr B. 36 y/o with food poisoning and severe diarrhea for the past 3 days C. 40 y/o with 3-day hx of chemo-induced V D. 75 y/o with pyelonephritis and hypoTN E. 82 y/o due for hemodialysis with clotted AV shunt

Answer: B, D, and E

What is atelectasis?

Collapse of the alveoli in the lung prevents normal exchange of O2 and CO2 - May be the result of airway obstruction caused by accumulated secretions or failure of the client to deep breathe or ambulate after surgery - Post-op complication that usually occurs 1-2 days after surgery

Borderline personality disorder

Condition marked by extreme instability in mood, identity, and impulse control.

Uterine tachysystole

Condition of excessively frequent uterine contractions during pregnancy. This is defined as 6 contractions in a 10 minute period.

COPD diet consists of...

Eat frequent, small, high-calorie meals to conserve energy.

What is a normal Mean Arterial Pressure (MAP)?

Greater than or equal to 90

What type of knot should restraint straps be tied?

In a quick-release knot (In case of an emergency)

TKO IV rate stands for

"To Keep Open" (Rate needed to keep the vein open)

1. What are the various classifications of shock? A. Septic B. Hypovolemic C. Anaphylactic D. Cardiogenic E. Neurogenic F. Obstructive 2. What is a normal serum lactate level?

1. A. Infection B. Fluid loss C. Allergic reaction D. Weak heart E. Neuro damage F. Impeding blood flow (commonly caused by cardiac tamponade) 2. Less than 1 mmol/L (*if serum lactate level is higher than this= anaerobic metabolism is occuring= BAD/ seen in shock!)

1. Types of female pelvis A. Gynecoid B. Anthropoid C. Android D. Platypelloid 2. Which one is the normal female pelvis?

1. A. Transversely rounded or blunt B. Oval shape; adequate outlet with a narrow pubic arch C. Heart shaped; resembles a male pelvis; NOT FAVORABLE for labor and vaginal birth; narrow pelvic planes can cause slow descent and mid pelvic arrest D. Flat with an oval inlet; wide transverse diameter, but short anteroposterior diameter, making labor and vaginal birth difficult 2. Gynecoid

What two times will a child receive the MMR vaccine?

1. 12-15 months 2. 4-6 years

HTN Classifications- 1. Pre HTN 2. Stage I HTN 3. Stage 2 HTN

1. 120-139/ 80-89 mmHg 2. 140-159/ 90-99 mmHg 3. over 160/ over 100 mmHg

What are the normal ABG values for... 1. pH 2. PaCO2 3. HCO3- 4. PaO2 (partial pressure of O2 dissolved in the blood) 5. SaO2 (O2 in blood bound to Hgb)

1. 7.35-7.45 2. 35-45 mmHg 3. 22-26 mEq/L 4. 80-100 5. 95-100%

1. Apathy definition 2. What sx is a common sigh of infection in the older adult, especially with a UTI?

1. A lack of interest or concern 2. Confusion

1. What is an ileus? 2. What is paralytic ileus? 3. What are some assessment findings?

1. A painful obstruction of the ileum or other part of the intestine. 2. Paralysis of the bowel due to surgery (common --especially in abdominal surgery). - Failure of appropriate forward movement of bowel contents - May occur as a result of anesthesia meds or manipulation of the bowel during the surgical procedure. 3. Vomiting post-op, abdominal distention, absent bowel sounds, bowel movement or flatus

Foreign body aspiration- What to in this scenario with a... 1. Child over 1 year of age (no longer an infant) 2. An infant 3. Should you blind sweep the childs mouth?

1. Abdominal thrusts (Heimlich manuever) - Applying upward thrusts to the upper abdomen just beneath the rib cage. 2. Back blows and chest thrusts 3. NO; this can force a loosely obstructing object to fully block the airway or cause the object to fall further into the airway, requiring surgical removal.

Place the nursing actions for performing a renal system physical assessment in the correct order. All options must be used. - Advise client to empty the bladder completely - Document the assessment of renal system function - Percuss and palpate both R and L kidneys - Observe skin and contour of abdomen and lower back - Auscultate the renal arteries in the R and L upper quadrants

1. Advise client to empty the bladder completely. 2. Observe skin and contour of abdomen and lower back. 3. Auscultate the renal arteries in the R and L upper quadrants. 4. Percuss and palpate both R and L kidneys. 5. Document the assessment of renal system function.

What are the 3 types of transmission? A. Diseases associated B. Barrier protection

1. Airborne A. Measles, varicella (chickenpox), TB B. Negative airflow pressure with high-efficiency particulate air (HEPA) filter mask - Workers wear a preparatory mask (N95) 2. Droplet A. Adenovirus, diphtheria, epiglottitis, influenza, meningitis, mumps, pneumonia B. Surgical mask - Mask on patient when they leave the room 3. Contact A. C diff, influenza B. Gloves and gown

Constipation is a general side effect from what classes of medications? (3)

1. Anesthetics (Ex= Propofol; Benzodiazepine like Midazolam) 2. Opioids 3. Anticholinergics

1. A nurse is preparing to administer a unit of packed red blood cells to a client with hemoglobin of 7 g/dL (70 g/L). The unit secretary retrieved the blood 25 minutes ago. When entering the client's room, the nurse notes that the client's IV is not patent and is unsuccessful at inserting the new IV. What should the nurse do next? A. Have another nurse attempt to restart the IV B. Notify the HCP of the delay C. Place the blood in the unit refrigerator D. Return the blood to the blood bank 2. How long can blood products be left at room T for before a transfusion?

1. Answer: D Not C= b/c blood products should be placed here as the T cannot be precisely regulated to decrease likelihood of bacterial growth. 2. Blood products should not be left at room T for greater than 30 min before a transfusion is started.

How to stop epistaxis

1. Apply direct and continuous pressure to the nasal bone for 10 mins (to promote clot formation) 2. Hold a cold cloth or ice pack to the bridge of the nose (to induce vasoconstriction and slow bleeding) 3. Keep the client quiet and calm (Clients should sit upright with the head forward)

Examples of non-steroidal anti-inflammatory medications?

1. Aspirin 2. Celebrex 3. Ibuprofen

How is Hepatitis B transmitted?

1. Blood 2. Semen 3. Vaginal secretions (Common through unprotected sex and IV illicit drug use) Note: *Hep A is transmitted in the fecal-oral route from improper food handling, therefore it is transmitted through the feces.

What rules are encompassed in Kosher dietary laws for Orthodox Jews? (2)

1. CanNOT consume pork, shellfish, and fish w/o scales. 2. Must separate meat/poultry from dairy. When meat/poultry is consumed, at least 3-6 hours must pass before a dairy product is consumed.

Peritoneal dialysis- 1. What is it? 2. What is the "dwell phase?" 3. The nurse is caring for a client whose peritoneal dialysis is beginning to exhibit insufficient outflow. What actions should the nurse perform initially? Select all that apply. A. Assess for abdominal distention and constipation B. Contact the HCP C. Examine the catheter for kinks and obstructions D. Flush the tubing with 100 mL of dialysate E. Place the client in a side-lying position

1. Catheter placed in the peritoneal cavity (abdominal lining) and dialysate is infused. 2. Tubing is CLAMPED to allow the fluid to remain in the cavity usually for 20-30 min= dwell phase - Tubing is then UNCLAMPED to allow dialysate to drain via gravity - Note: insufficient outflow results from CONSTIPATION 3. Answer: A, C, and E

What are the two signs/ sx's of hypocalcemia?

1. Chvostek's sign: Contraction of facial muscles in response to light tap over facial nerve in front of the ear. 2. Troussau's sign: Carpal spasm induced by inflating BP cuff.

What are the 4 types of adventitious lung sounds?

1. Crackles - Types include fine, medium, and coarse 2. Wheeze - Similar to a "squeak" - From a narrowed airway like in asthma 3. Rhonchi - Low snoring or moaning tone - Obstruction of the trachea or bronchus (upper airway from secretion build up) 4. Pleural friction rub - Sounds like 2 surfaces rubbing together

What are the types of delusions? (6)

1. Delusions of reference (believe that songs, newspaper articles, and other events are personal and significant to them). - Ex= "That song is a message sent to me in a secret code" 2. Persecutory (paranoid) delusion: believe that they are being threatened or treated unfairly in some way. - Ex= "those Martians are trying to poison me with the tap water" 3. Delusions of Grandeur: false impression of one's own importance. - Ex= I need to get to Washington for my meeting with the president" 4. Delusion of control: false belief that another person, group of people, or external force CONTROLS one's general thoughts, feelings, impulses, or behavior. - Ex= "Don't drink the tap water, Thats how the government controls us" 5. Nihilistic delusion: a delusion that things (or everything, including self) do NOT exist. A sense that everything is unreal. - Ex= "It doesn't matter if I take my medicine. Im already dead." 6. Somatic delusion: false belief that one's bodily functioning, sensation, or appearance is grossly abnormal. Thinking that your body is diseased in some way.- - Ex= "The doctor said Im fine, but I really have lung cancer."

Describe the 6 Stages of Change for successful behavior modification/ readiness and motivation to change- 1. Precontemplation 2. Contemplation 3. Preparation 4. Action 5. Maintenance 6. Termination

1. Does not believe a problem exists. 2. Undecided whether it would be possible or worthwhile. 3. Begins establishing goals. 4. Actively takes steps toward new behavior. 5. Preventing relapse. 6. Desired change achieved and relapse to formal behaviors always possible.

What are the 3 bypasses in fetal circulation?

1. Ductus arteriosus: connects the pulmonary artery to the aorta to bypass the lungs. 2. Ductus venosus: comments the umbilical vein and the inferior vena cava to bypass the liver. 3. Foramen ovale: opening between the R and L atria of the heart to bypass the lungs.

How often do you change... (according to CDC recommendations) 1. IV site dressing? 2. IV tubing?

1. Every 72-96 hours (3-4 days) 2. Every 96 hours (4 days)

Positions- 1. Lithotomy position 2. Sims position

1. For female genitalia and reproductive tract exams - Supine position with legs separated (child birthing position) 2. Patient laying down on left side and right hip/knee bent (on stomach= aka how you sleep) - For rectal exams, treatments or enema administration

What are the common causes of metabolic acidosis? (5)

1. GI bicarbonate losses (diarrhea) 2. Ketoacidosis (DM, ETOHism, starvation) 3. Lactic acidosis (sepsis, hypo perfusion) 4. Renal failure (hemodialysis with inaccessible AV shunt) 5. Salicylate toxicity (ASA toxicity) *Note: Metabolic alkalosis would result from vomiting b/c of loss of stomach acid.

What are the various kinds of Fowlers positions: 1. Low Fowlers 2. Semi-Fowlers 3. Standard Fowlers 4. High Fowlers (Describe each)

1. HOB 15-30 degrees 2. HOB 30-45 degrees 3. HOB 45- 60 degrees 4. HOB 60-90 degrees

Teaching points to assist a client in appropriate use of a cane: (5) *Note at the end is really helpful

1. Hold the cane on the STRONGER SIDE to provide maximum support and body alignment. Keeping the elbow slightly flexed (20-30 degrees). 2. Place the cane 6-10 inches in front of and to the side of the foot to keep the body weight on both legs to provide balance. 3. For maximum stability, move the weaker leg forward to the level of the cane, so that body weight is divided between the cane and stronger leg. 4. Move the stronger leg forward past the cane and weaker leg, so that weight is divided between the cane and weaker leg. 5. Always keep at least 2 points of support on the floor at all times. *Note: You first want your cane to be the right height (measured from your greater trochanter to the ground with elbow at a 20-30 degree angle) - Hold the cane on the OPPOSITE side of your INJURED LEG - Cane will ALWAYS follow the injured leg (aka if injured foot goes forward, the cane goes forward and vice versa).

The nurse is performing a physical assessment on a 2-year old with cold sx and a fever of 101.7F. The parent is concerned about the child's ability to cooperate during the examination. Place the components in the order the nurse would perform them. All options must be used. (1-5) - Measure the childs height and weight - Play with the child using a finger puppet - Take the child's VS - Interact with the parent in a friendly manner - Auscultate the childs heart and lungs

1. Interact with the parent in a friendly manner 2. Play with the child using a finger puppet 3. Measure the childs height and weight 4. Auscultate the childs heart and lungs 5. Take the child's VS

What are the three categories of food that are rich in iron?

1. Meats (beef, lamb, chicken, pork, liver). 2. Shellfish (shrimp, oysters, clams). 3. Eggs, green leafy veggies, broccoli, dried fruit, dried beans, brown rice, oatmeal.

Brudzinski and Kernig's signs- 1. What do they both indicate if positive? 2. How to do Brudzinski sign test 3. How to do Kernig sign test

1. Meningeal irritation (for meningococcal meningitis) 2. Patient in supine position. RN flexes head to chest and no pain should be reported. 3. Severe stiffness of hamstrings causes inability to straighten (extend) the leg when hip is flexed to 90 degrees.

What foods are indicated in a high-iron diet?

1. Organ meats 2. Meat 3. Egg yolks 4. Whole-wheat products 5. Dark green leafy veggies 6. Dried fruit 7. Legumes (Inform client to concurrently take Vit C with iron foods to enhance absorption)*

What are some examples of uterotonic medications used in postpartum uterine atony? (4)

1. Oxytocin 2. Methylergonovine 3. Carboprost 4. Misoprostol

The nurse caring for a male client prepares to insert an indwelling urinary catheter. The nurse assesses for allergies, explains the procedure to the client, and asks unlicensed assistive personnel to perform perineal care while equipment is gathered. Place in order the steps the nurse should take when inserting the urinary catheter. All options must be used. - Advance catheter to tubing bifurcation and inflate balloon - Perform hand hygiene and apply sterile gloves - Place penetrated drape with shiny side DOWN - Use dominant hand to cleanse meatus with cotton balls or swab sticks - Use dominant hand to insert catheter until urine return is observed - Use non dominant hand to grasp penis below glans

1. Perform hand hygiene and apply sterile gloves. 2. Place penetrated drape with shiny side DOWN. 3. Use non dominant hand to grasp penis below glans. 4. Use dominant hand to cleanse meatus with cotton balls or swab sticks (cleanse from meatus to glans in a circular motion). 5. Use dominant hand to insert catheter until urine return is observed. 6. Advance catheter to tubing bifurcation and inflate balloon.

The nurse is teaching a 9 y/o child with asthma how to use a metered-dose inhaler. Place the instruction in appropriate order. - Rinse mouth with water - Shake MDI and attach it to spacer - Place lips tightly around the mouth piece - Exhale completely - Deliver one puff of med into spacer - Take a slow deep breath and hold for 10 secs

1. Shake MDI and attach it to spacer. 2. Exhale completely to optimize inhalation of the med. 3. Place lips tightly around the mouth piece. 4. Deliver one single puff of med into spacer. 5. Take a slow deep breath and hold for 10 secs to allow for effective med distribution. 6. Rinse mouth with water to remove any left over med from oral mucous men's. Spit out water to ensure no med is swallowed.

Match the services to the job in the multidisciplinary team- 1. Communication, speech, and swallowing/eating issues 2. Help with developing coping skills, securing adequate financial resources or housing, and making referrals to volunteer organizations 3. Assessing and planning optimal care of any wound 4. Help with achieving skills necessary for ADLs 5. Walker training, ability to transfer, and mobility Options: wound care, speech therapy, PT, OT, and social worker

1. Speech therapy 2. Social workers 3. Wound care 4. OT 5. PT

What steps to do when a blood transfusion reaction occurs?

1. Stop the transfusion. 2. Change IV tubing down to the IV site and keep the line open with NS. 3. Notify HCP and blood bank. 4. Stay with the client, observing s/s and monitor VS q 5 mins. 5. Prep to admin emergency meds as prescribed. 6. Obtain urine specimen for lab studies. 7. Return blood bag, tubing, attached labels, and transfusion record to the blood bank. 8. Document the occurrence, actions taken, and client's response.

What are the two major issues associated with HYPOMg2+?

1. Ventricular arrhythmias (Torsades des pointes) - this is the most SERIOUS concern (priority) 2. Neuromuscular excitability (manifestations similar to those found in hypoCa2+) - Includes tremors, hyperactive reflexes, seizures, and positive Trousseau and Chvostek signs

Osteogenesis imperfecta

A rare genetic condition resulting in impaired synthesis of collagen by osteoblasts (cells that build bone). Impaired collagen causes bones to be frail and easily fractured. Usually transmitted by autosomal dominant inheritance.

Delirium

A serious disturbance in mental abilities that results in confused thinking and reduced awareness of the environment. The start of delirium is usually rapid (within hours or a few days).

What is the obturator function on a tracheostomy?

A stylet with a smooth end used to facilitate the direction of the tube when inserting or changing a tracheostomy tube. - Device that replaces the inner cannula temporarily during insertion of the trach, then after the insertion take out the obturator and replace it with the inner cannula. - *The obturator is removed immediately after tube placement and is always kept with the patient at the bedside in case of accidental decannulation.

A. Alcohol withdrawal sx's (11) B. Delirium tremens and other withdrawal sx's can be prevented with what medications administered?

A. 1. Agitation 2. Hyperreflexia (involuntary nervous system overreacts to external or bodily stimuli) 3. Insomnia 4. Anxiety 5. Tremors 6. Diaphoresis 7. Palpitations 8. GI upset 9. SEIZURES (can be single or multiple generalized tonic-clonic) 10. Alcoholic hallucinosis (visual, auditory, tactile) 11. DELIRIUM TREMENS (hallucinations included) B. Benzodiazepine administration - Alprazolam (Xanax) - Clonazepam (Klonopin) - Chlordiazepoxide (Librium) - Diazepam (Valium) - Lorazepam (Ativan)

A. Negative sx's of schizophrenia (10) B. Positive sx's of schizophrenia

A. 1. Impaired social and interpersonal functioning 2. Inappropriate or blunt affect 3. Apathy (lack of enthusiasm/ concern) 4. Inadequate motivation 5. Inability to make decisions 6. Poor personal hygiene 7. Bizarre posturing 8. Pacing and rocking 9. Regression 10. Inability to experience pleasure B. 1. Delusions 2. Hallucinations

A. How many arteries and veins are in the umblical cord? B. What kind of blood do each carry?

A. 2 arteries 1 vein B. Arteries (2) carry DEOXYGENATED blood from baby to mom Vein (1) carry OXYGENATED blood from mom to fetus

Malignant hyperthermia- A. What is it? B. Where is it commonly seen? C. What are the s/s? D. What is the tx?

A. A hereditary condition of uncontrolled heat production that occurs when susceptible people receive certain anesthetic drugs. - This is rare, life-threatening - Inherited muscle abnormality that is triggered by certain drugs used to induce general anesthesia in susceptible clients - The triggering agent leads to excessive release of Ca2+ from the muscles, leading to sustained muscle contraction and rigidity B. Can occur in the OR or in the PACU C. 1. Hypercapnia (earliest sign) 2. Generalized muscle rigidity 3. Hyperthermia (later sign) D. Nurse would notify the HCP, indicating the need for immediate tx - DANTROLENE* (a muscle relaxer) - cooling blanket - fluid resuscitation

A. What is Chvostek and Trousseau signs? B. What lyte imbalances cx this?

A. Chvostek Sign= when tap facial nerve, the face muscle twitches Trousseau Sign= carpopedal spasm (hand cramps up) B. Seen in low Mg2+ and low Ca2+

A What are the atypical antipsychotics? B. What do they tx? C. What are some major side effects in taking atypical antipsychotics? D. What are the typical antipsychotics?

A. Risperidone Clozapine Olanzapine Quetiapine Aripiparzole Ziprasidone B. Schizophrenia, BP d/o, and other mental health d/o C. 1. Extrapyramidal sx's - Akathisia= restlessness; fidgeting - Parkinsonism= tremors; shuffling gait 2. Neuroleptic malignant syndrome 3. Anticholinergic effects - Dry mouth - Constipation 4. Orthostatic hypoTN 5. Sedation - Drowsiness - Hypersomnia= excessive sleeping (common)*

A. What is the difference between wound dehiscence and wound evisceration? B. How should the nurse position the client with the eviserated wound while waiting to cover the wound? C. What kind of dressing should the wound be covered with while waiting for the MD to return the client to surgery?

A. Wound dehiscence: separation of the wound edges at the suture line. See appearance of underlying tissues through the wound. Wound evisceration: protrusion of the internal organs through an incision. Appearance of loose of bowel or other abdominal contents through the wound. This is an emergency!* B. Place in Low Fowler's position with knees bent C. Sterile normal saline dressings

Trismus- A. Also known as... B. What is it?

A. "Lock jaw" B. The inability to open the mouth due to tonic contraction of the muscles used for chewing. - May indicate a more serious complication of tonsillitis

A. Myopia B. Hyperopia

A. "Nearsightedness" (can see near but not far) - Image focuses in front of the retina - Reduced visual acuity when viewing objects at a distance B. "Farsightedness" (can see far but not near) - Image focuses behind the retina

Tetralogy of Fallot A. What are the 4 structural defects in this condition? B. What are the clinical sx's of Tetralogy of Fallot? C. What actions should the nurse include to reduce the incidence of hyper cyanotic spells?

A. "RAPS" R= right ventricular hypertrophy A= aorta displacement P= pulmonary stenosis S= septal defect (ventricular) B. "Afflict" A= activity (cyanosis, SOB, increased RR= "tet spell") F= finger nail changes (clubbing from chronic hypoxia over 6 months) F= fatigue/faints easily (reduced O2/ activity intolerance) L= lift knee to chest or squats I= inability to grow C= cardiac sounds (systolic murmur and thrill) T= trouble feeding C. 1. Encourage smaller, frequent feedings 2. Offer a pacifier when the infant begins to cry 3. Promote a quiet period upon waking in the morning 4. Swaddle the infant during procedures

Adult chest compressions- A. How many compressions per rescue breaths? B. How deep to provide high-quality chest compressions during CPR? C. What is the recommended rate? D. How should you place your hand? E. CPR compressions are paused every _____ mins to assess the clients pulse, but this pause should be no LONGER than _____ secs to minimize delays between compression cycles.

A. 30 compressions followed by 2 rescue breaths B. 2-2.4 in (5-6cm) C. 100-120/min D. Center of the chest on the lower half of the sternum (breastbone) E. 2 mins; 10 seconds

Autonomic dysreflexia- A. What is it? B. What is the common cause? C. What are the common assessment findings? D. What are tx interventions?

A. A massive, uncompensated cardiovascular reaction by the sympathetic nervous system (SNS) in a spinal injury at T6 or higher. - There is a stimulus sensing discomfort below the level of the injury = ANS sends SNS signals and vessels vasoconstrictor below the level of the injury - Baroreceptors above the level of injury are trying to send a signal to decrease HR= Vasodilation/ decreased HR above level of injury; vasoconstriction below the level of injury B. Distended bladder! - Another one is a full/ impacted bowel C. HA, nasal stiffness, flushed face, sweating D. 1. Raise HOB to relieve pressure in the head from the vasodilation 2. Find pain stimulus 3. Meds to decrease BP

A. What is referring syndrome? B. What are the key signs of referring syndrome? (PPM mnemonic)

A. A potentially lethal complication of nutritional replenishment in significantly malnourished clients. After a period of starvation, carb-rich nutrition (glucose) stimulates insulin production along with a shift of lytes from the bl into tissue cells for anabolism. B. Rapid declines in... P- Phosphorus P- Potassium M- Magnesium Other findings may include: - Thiamine deficiency - Hyperglycemia - Na+ retention - Fluid overload

Conduct disorder- A. What is it? B. What are the sx's? C. What is the tx?

A. A serious behavioral and emotional disorder that can occur in children and teens. A child with this disorder may display a pattern of disruptive and violent behavior and have problems following rules. B. 1. Aggressive behavior (fighting, bullying, animal cruelty, using weapons, rape) 2. Destructive behavior (intentional destruction of property--setting fire/arson and vandalism) 3. Deceitful behavior (repeated lying, shoplifting, or stealing) 4. Violation of rules C. 1. Psychotherapy (type of counseling) 2. Some medications used off-label to tx distressing sx's (impulsivity, aggression) as well as any other mental illness (ADHD or MDD)

Epiglottitis- A. What is it? B. What are common clinical manifestations? C. Nursing interventions? D. How can you educate the parent to PREVENT this condition?

A. A sudden-onset MEDICAL EMERGENCY due to Haemophilus influenzae, causes severe inflammatory obstruction above and around the glottis. B. Child will typically progress from having no sx to having a completely occluded airway within hours. - Sitting in a TRIPOD position is a classic presentation - DROOLING - Restlessness and anxious from the airway obstruction and hypoxia - Stridor (inspiratory) - Increased T (HIGH) - Difficulty speaking C. Throat inspection should NOT be done until emergency intubation is readily available - NEVER insert ANYTHING in the patients mouth to assess it! (no tongue blades, oral T, or cultures as this can cx spasms that further obstruct the airway) - Crash cart with intubation equipment near by - Keep pt NPO; admin antibiotics; IV fluids; and corticosteroids D. Get their child VACCINATED - Have the HIB vaccine administered - Child would need 3-4 doses at 2, 4, 6, and 12-15 months of age

Confirm the form of isolation for the following situations. (Pick airborne, droplet, or contact precautions) A. Measles B. MRSA C. Rapid Influenza D. Scabies E. Tuberculosis F. Varicella (chicken pox) G. What form of isolation takes priority over the other types?

A. Airborne B. Contact C. Droplet D. Contact E. Airborne F. Airborne G. Airborne isolation is first priority, then droplet, then contact (b/c infectious agents that are spread by air currents are among the most contagious pathogens)

A. When is the neonates apgar score taken? B. What does it indicate? C. What does an apgar score of 10 indicate versus a score of 3? D. What 5 components in "APGAR" do you assess?

A. At 1 and 5 minutes after birth B. Indicates how the neonate is doing after birth and managing extrauterine life. C. Score of 10= best possible condition Score of 3= immediate resuscitation D. 1. Appearance/color 2. Pulse 3. Grimace 4. Activity 5. Respiratory effort

Probable (objective) signs of pregnancy: A. Chadwick's sign B. Goodall's sign C. Hegar's sign D. Braxton-Hick contractions

A. Blue/purple color of the cervix, vagina and vulva B. Softening cervix C. Softening uterus (Goodell "gooey") (Hegar's "hands" because when the health examiner examines the uterus, he/she puts one hand in the uterus and other on top to feel gooey uterus during pregnancy) D. Early contractions that are NOT TRUE contractions because it is not changing the cervix (*A positive pregnancy test is another PROBABLE change of pregnancy!)

What are the common physical manifestations for the end of life? A. Sensory B. Respirations C. Circulatory D. Elimination (urinary and GI) E. Musculoskeletal

A. Blurred vision, decreased sense for taste and smell, appears to stare (*hearing is the last sense lost) B. Rapid or slow, maybe noisy/wet= "death rattle", CHEYNE-STOKES respirations (alternating periods of apnea and deep/rapid breathing C. Reduced HR and BP progressively, skin is cool, pale, mottled (blotchy, red-purplish marbling of the skin usually occurring before death or final week of life), and cyanotic D. Decreased u/o, increased incontinence, reduced GI and peristalsis= constipation, gas accumulation and distention motility E. Reduced ability to move, dysarthria (difficulty speaking), dysphagia (difficulty swallowing), reduced gag reflex

What symptoms correlate to which blood transfusion reaction? List of blood transfusion reaction types (allergic, anaphylactic, febrile non-hemolytic, acute hemolytic, circulatory overload, septicemia, transfusion-related lung injury) A. Cough, SOB with coarse crackes, ↑HR, ↑BP, neck vein distension (JVD) B. Acute onset fever, chills, progressive respiratory distress, cardiac symptoms C. Urticaria (hives), flushing, itching D. Fever +/- pulmonary symptoms, sudden chills and fever (~1◦C, 1.8◦F), headache, flushing E. Chills, fever, low back pain, tachycardia, tachypnea, hematuria, ↓BP, ARF (acute renal failure) F. Chills (rapid onset), high fever, diarrhea, marked ↓BP (hypotensive) G. Urticaria, wheezing/obstruction in bronchioles, anxiety, shock

A. Circulatory overload B. Transfusion-related lung injury C. Allergic D. Febrile, non-hemolytic E. Acute hemolytic F. Septicemia G. Anaphylactic

A. If PTT is LOW, then do you increase or decrease the Heparin infusion? B. What about if PTT is higher than 60-70 seconds?

A. FOR LOW PTT: Increase the Heparin infusion! B/c the clotting time is faster than normal. B. FOR HIGH PTT: Decrease the Heparin infusion! B/c the clotting is not fast enough now from the normal= bleeding risk!*

Define- A. Cholelithiasis B. Acute cholecystitis

A. Gallstones B. Inflammation of the gallbladder.

Cystic fibrosis A. What is the recommended diet? B. What long-term complication is often associated with CF?

A. High calorie, high protein, and high fat B. Recurrent pneumonia

A. The presence of an air leak in a chest drainage system is located in... B. Where would the nurse expect gentle, continuous bubbling in a client with with a pneumothorax?

A. Indicated by continuous bubbling of fluid at the base of the water seal chamber. B. Suction control chamber

Infective endocarditis- A. What is it? B. Common cause? C. Symptoms? (10) D. Treatment

A. Inflammation of endothelium that lines heart and cardiac valves. -Most commonly damages mitral valve, then aortic and tricuspid valves. B. Commonly caused by bacteria that are normally present in the body. - Can also occur after an invasive medical or dental procedure. C. 1. Valvular dysfunction 2. Chest pain 3. CHF 4. Clubbing 5. Meningitis 6. Low back pain 7. Arthralgia (joint pain) 8. Arthritis (inflamed joints) 9. SPLINTER HEMORRHAGES can occur with infection of the heart valves 10. Fever D. IV Antibiotics for 4-6 weeks

Meningococcal meningitis- A. What is it? B. How is it spread? What kind of precautions are implemented? C. What are the clinical manifestations? (classic triad of sx's + others) D. Diagnosis includes which two signs? E. What is the tx?

A. Inflammation of the meninges (inner two membranes around the brain and spinal cord). B. Hematogenous--pathogen gets into the blood vessels and moves through endothelial cells in the vessels of the blood brain barrier (once the WBCs are ID this causes inflammation within the CSF) Droplet precautions (nurse should wear a mask when caring for the client) C. Classic Triad: 1. Fever 2. Nuchal rigidity (neck stiffness) 3. Headache Others include- 4. Photophobia (discomfort with bright lights) 5. Photophobia (discomfort with loud sounds) D. 1. Kerning's Sign: knee at 90 degree angle, when straighten leg if pain in back then positive sign. 2. Brudzinski's Sign: on back with neck flexed up and supported, if knees/hips flex then this is a positive sign (Another form of diagnosis is a lumbar puncture- for this procedure, have the clients head and knees tucked in and the back rounded out) E. Steroids then antibiotics/ antivirals/antifungals/antiparasitics

A. What is the first line anti tubercular drug that is used to treat a latent TB infection? B. What are 2 serious side effects?

A. Isoniazid (INH) B. 1. Hepatotoxicity (ex= scleral and skin jaundice, vomiting, dark urine, fatigue) 2. Peripheral neuropathy (ex= numbness, tinging of extremities)

A. What is Cushing's Triad symptoms of? B. What are the 3 components to Cushing's Triad?

A. Late symptoms of increased intracranial pressure (ICP) B. 1. Bradycardia 2. Slowed/ altered RR 3. Widened pulse pressure (increased SBP)

A. Normal troponin level B. What does it indicate when troponin is elevated?

A. Less than 0.1 B. Indicative of cardiac DAMAGE

Disaster triage types and characteristics of each: A. Emergent (red) B. Urgent (yellow) C. Nonurgent (green) D. Expectant (black)

A. Life threatening injuries with high probability for survival if immediate tx is received. B. Serious injuries requiring tx within 30 min and 2 hr. C. Injuries requiring tx but can wait for 2 hr or longer. D. Extensive injuries with poor prognosis regardless of tx (near death).

What is... A. aphasia B. Function of Wernickes area of the brain C. Function of Brocas area of the brain

A. Loss of ability to understand or express speech, caused by brain damage. B. Helps in speech processing and UNDERSTANDING language. C. Helps in PRODUCING coherent speech.

A. What is homonymous hemianopsia? B. What should be taught to avoid self-neglect?

A. Loss of half of the field of view on the same side in both eyes Seen in clients following a cerebral vascular accident (stroke). Risk for self-neglect on the affected side can occur. B. These clients are at higher risk for injury b/c they are unable to incorporate full visual field input. They are taught to turn the head and scan to the side with the visual field deficit to reduce the risk for injury and self-neglect.

Types of impaired thought processes seen in individuals with Schizophrenia- A. Neologisms B. Concrete thinking C. Loose associations D. Echolalia E. Tangentiality F. Word salad G. Clang associations H. Perservation

A. MADE-UP words or phrases usually of a bizarre nature; the words have meaning to the client only. Ex= "I would like to have a phijnox" B. Literal interpretation of an idea; the client has difficulty with abstract thinking. Ex= "The grass is always greener on the other side" is interpreted to mean that the grass is literally greener. C. Rapid shifting from one idea to another, with little or no connection to logic or rationality. Ex= The world turns as the world turns on a ball at the beach. But all the world's a stagecoach and I took the bus home." D. Repetition of words, usually uttered by someone else. E. Going from one topic to the next without getting to the point of the original idea or topic. F. A mix of words and/or phrases having no meaning except to the client. Ex= "Here what comes table, sky, apple." G. Rhyming words in a meaningless, illogical manner. Ex= "The pike likes to hike and Mike fed the bike near the tyke." H. Repeating the same words or phrases in response to different questions.

Cerebral perfusion pressure- A. What is the equation?*

A. Mean arterial pressure (MAP)- Intracranial pressure (ICP)

A. What is parallel play? B. What is associative play? C. What is cooperative play?

A. Mimic the activities of others, a kind of learning play. This is typical behavior of a TODDLER. Playing alongside, rather than with, other children. - Ex's of parallel play: pushing and pulling large toys; smearing paint; playing with dolls or toy cars; and digging in a sandbox B. Start to become interested in joining others in their play activities. This is typical behavior of a preschooler. C. Common in school-age children. These children play with one another with a specific goal, often with in a rigid set of rules.

A. FLACC scale is used to B. What does FLACC stand for/ what are you assessing?

A. Nonverbal signs of pain B. Face Legs Activity Cry Consolability (The lower the # on the FLACC scale, the less pain the client is in)

A. During bouts of acute diarrhea and dehydration, tx focuses on maintaining adequate fluid and lyte balance through what first-line tx? B. Can you administer antidiarrheal medications to a child?

A. Oral rehydration therapy B. Antidiarrheal medications is discouraged in children b/c they have little effect in controlling diarrhea and may be harmful by prolonging some bacterial infections and casing fatal paralytic ileus in children.

Strabismus- A. What is the intervention for strabismus? B. What is esotropia? C. What is exotropia? D. What is amblyopia? E. What is diplopia?

A. Patching of the stronger eye with progressive improvement. B. 1 eye deviated inward C. 1 eye deviated outward D. "lazy eye" E. brain perceives 2 images

Duchenne muscular dystrophy- A. What is it? B. Clinical manifestations? - What is the Gower maneuver? C. Tx

A. Progressive muscular weakness, wasting and contractures (slowly progressive, generalized weakness during teenage years) B. 1. Calf muscle pseudo hypertrophy 2. Walk on tiptoes 3. Frequent tripping and falling - Gower maneuver: placing hands on the thighs to push up to raise themselves to stand C. No cure - Maintain optimal function in muscles as long as possible - Prevent contractors

A. What stage of labor can a systemic analgesia may be administered? B. Why would an opioid agonist-antagonist medication need to be given at that specific stage? C. What are the 2 commonly used opioid agonist-antagonist medications used in labor?

A. Stage 1- Active phase (When the mom is having well-established labor contractions, cervix dilated 4-5cm, and FHR 110-160 bpm/maternal VS are stable) B. B/c these medications also cross the placental barrier resulting in an effect on the fetus depending on the time of administration prior to delivery. C. Butorphanol tartrate (Stadol) and Nalbuphine hydrochloride (Nubain)

Glasgow Coma Scale- A. What does it assess? (aka what 3 categories) B. What is the maximum score that you could get? (What does that mean?) C. What does a GCS under 8 mean?

A. The GCS quantifies the level of consciousnessness in a client with acute brain injury. - This is done by measuring: 1. Eyes (alertness) 2. Verbalresponse (orientation) 3. Motor response (ability to obey commands/ frontal lobe functioning) B. Maximum of 15 points; this implies that the client is FULLY CONSCIOUS= GOOD! C. Below 8= coma; the lowest it can go is a 3

Electroconvulsive therapy (ECT) A. What is it? B. What measures are taken to minimize seizure and prevent musculoskeletal injury? C. What does it tx? D. What are the 2 COMMON SIDE EFFECTS? E. What is the usual course of acute therapy for ECT? F. Should a client take Valproic acid prior to the procedure?

A. The client is tx with pulses of electrical energy through electrodes applied to the scalp. The electrical stimulus is sufficient to cause a brief convulsion! B. General anesthesia and a skeletal muscle relaxant. C. Severe major depressive disorder (when clients pose a severe threat to themselves and it is not safe to wait until medications take effect). D. #1- Confusion #2- Memory loss E. 6-12 ECT treatments performed 2-3 times a week. F. NO; b/c Valproic acid is an anticonvulsant prescribed for bipolar d/o; therefore it would prevent the therapeutic effect of the ECT inducing a generalized seizure.

A. What is pyloric stenosis? B. What laboratory value should the nurse expect if the client has suspected pyloric stenosis? (1 + what ABG reading) C. What are the clinical sx's?

A. The gradual hypertrophy of the pylorus until sx onset at 3-5 weeks of life. B. Elevated Hct (due to the dehydration from the vomiting); ABG= metabolic alkalosis C. 1. Postprandial (after meal) PROJECTILE vomiting followed by hunger - Vomit is non bilious (b/c obstruction is proximal to the bile duct) 2. Poor weight gain 3. Dehydration (s/s include sunken fontanel, reduced skin turgor, and delayed cap refill)

Esophageal atresia and tracheoesophageal fistula (EA/TEF) A. What is it? B. Clinical manifestations include? C. Which nursing interventions should be included in the plan of care for a newborn with suspected EA and TEP? Select all that apply. 1. Keep the infant NPO 2. Maintain the infant supine with HOB 30 degrees 3. Place suction equipment by the infant's bed 4. Prepare for urgent gastrostomy tube placement to start feedings 5. Refer family to palliative care team

A. The upper esophagus ends in a blind pouch ad the lower esophagus connects to the primary bronchus or the trachea through a small fistula. B. Frothy saliva, choking, coughing, and drooling. Clients may also develop apnea and cyanosis when feeding. C. 1, 2, and 3 Not option 4 b/c would want to surgically correct the TEF before feedings or irrigations in the GI tract. Parenteral nutrition for the infant would likely be required prior to surgery.

Hyperemesis gravidarum- A. What is it? B. What are the clinical features? C. What are the laboratory abnormalities associated with the condition?

A. This is severe, persistent N and V during pregnancy. B. Weight loss, poor skin turgor, dry mucous membranes, hypoTN, and tachycardia C. hypoK+/ hypoNa+, Ketonuria (indicates that the body is breaking down fat to use for every our to the clients starvation state), increased urine specific gravity, hemoconcentration, metabolic alkalosis

Nephrotic syndrome- A. What is the cause? B. What are the clinical manifestations? C. Nursing interventions D. Risk for relapse?

A. Unknown (idiopathic) reason for autoimmune dz causing changes to the glomerulus= leaking of MASSIVE PROTEINS (like albumin)** B. 1. Proteinuria causing frothy, foamy, dark yellow urine 2. Hypoalbuminemia: - Hyperlipidemia - Edema (weight gain in the face, extremities, ascites/abdominal swelling). Edema is the result of reduced albumin levels in the blood b/c of its main role in regulating oncotic pressure. If no albumin in the blood, H2O will go out into the tissues. C. 1. Monitor fluid status and swelling 2. Corticosteroids or immune suppressors 3. Risk for infection (especially in swollen areas) 4. Assess for DVT, PEs (respiratory status) 5. Diet (decrease Na+/ fluids/ fats) D. Risk for relapse

Developmental Dysplasia of the Hip- A. What is it? B. What is the most common clinical sx? C. What is the most common and effective tx?

A. When a baby's hip is loose or dislocated from ball in socket joint not forming normally. B. Uneven skin folds when extending both legs. More folds on the affected side. C. Pavlik harness (a soft brace that stabilizes and correctly aligns the leg) - Worn 24 hr/day for 12 weeks under the clothes.

S/s of abrupt placentae and placenta previa

Abrupt placentae = "DETACHED" D= dark red bleeding E= extended fundal height T= tender uterus A= abdominal pain/ contractions C= concealed bleeding internally in most cases H= hard abdomen E= experienced D= DIC (massive blood clotting in the body which leads to clotting factors depleted) Placenta previa= "PREVIA" P= painless bright red bleeding from the vagina R= relaxed soft NON-TENDER uterus E= episodes of bleeding V= visible bleeding I= intercourse post-bleeding A= abnormal fetal position

How do you treat Postpartum hemorrhage?

Aggressive tx because this is an obstetric emergency. - Maintain adequate circulating volume (IV fluids and blood products -Includes ID the underlying cause and treating it. Fluid replacement, oxytocin intravaginal/rectal prostaglandins, uterine massage

What is the role of aldosterone in regulating H2O and Na+?

Aldosterone that the secreted by the adrenal glads controls ECF by regulating the amount of H2O reabsorbed by the kidneys by conserving Na+, recreation of K+ and retain to stabilize BP. *In cases of low BP, adrenal glands stimulated to release aldosterone in the blood stream to conserve Na+ so H2O will follow after. Doing this will increase BP to the normal level.

List of Tricyclic Antidepressants

Amitriptyline Amoxapine Desipramine Doxepin Imipramine Nortriptyline Protriptyline Trimipramine

What is Guillain-Barre Syndrome (GBS)?

An acute inflammatory demyelinating polyneuropathy characterized by rapidly progressive ascending symmetric motor weakness. It is immune mediated following infection. - It is often accompanied by ASCENDING muscle weakness and absent deep-tendon reflexes - Respiratory failure is the most life-threatening complication

A client rings the call light and complains of pain at the site of an intravenous (IV) infusion. The nurse assesses the site and determines that phlebitis has developed. The nurse should take which actions in the care of this client? Select all that apply. 1. Remove the IV catheter at that site. 2. Apply warm moist packs to the site. 3. Notify the health care provider (HCP). 4. Start a new IV line in a proximal portion of the same vein. 5. Document the occurrence, actions taken, and the client's response.

Answer: 1, 2, 3, and 5

The nurse provides a list of instructions to a client being discharged to home with a PICC. The nurse determines that the client needs more further instructions if the client made which statement? 1. "I need to wear a MedicAlert tag or bracelet" 2. "I need to restrict my activity while this catheter is in place" 3. "i need to keep the insertion site protected when in the shower or bath" 4. "I need to check the marking on the catheter each time the dressing is changed"

Answer: 2

The nurse is caring for a client who has deep venous thrombosis and is prescribed a continuous IV infusion of heparin 25,000 units in 500 mL of D5W at 1300 units/hr. After 6 hours of the heparin infusion, the client's PTT is 44 seconds. The nurse must adjust the infusion rate according to the heparin drip protocol (shown in the exhibit). According to the protocol, at what rate in milliliters per hour (mL/hr) should the nurse set the IV infusion pump? Click on the exhibit button for additional information. Record your answer using a whole number.

Answer: 28

Rule of Nines (Burns)- A 35 year old female patient has deep partial-thickness burns on the front and back of both arms, anterior trunk, back of left leg, anterior and posterior sides of the right leg, posterior head and neck, and perineum. What is the total body surface area percentage that is burned?

Answer: 9+9+18+9+18+4.5+1= 68.5% FRONT and BACK of both arms: 18% ANTERIOR trunk: 18% BACK of left leg: 9% ANTERIOR and POSTERIOR of right leg: 18% POSTERIOR head and neck: 4.5% Perineum: 1% Total= 68.5%

A client postoperative from a transurethral prostatectomy has a triple-lumen, indwelling urinary catheter and is receiving continuous bladder irrigation of sterile normal saline solution at 175 mL/hr. The nurse empties the urine drainage bag for a total of 2300 mL at the end of the 8-hour shift. How many milliliters (mL) should the nurse document as the net urine output for the shift? Record your answer using a whole number.

Answer: 900mL net urine output To calculate the net urine output for a client receiving CBI you need to subtract the irrigation input from the total catheter output.

The client has increased intracranial pressure with cerebral edema, and mannitol is administered. Which assessment should the nurse make to evaluate if a complication from the mannitol is occurring? A. Auscultate breath sounds to assess for crackles B. Monitor for greater than 50 mL/hr urine output C. Monitor GCS increasing 8/15 to 9/15 D. Press over the tibia to assess for pitting edema

Answer: A

The nurse assesses several clients using the Glasgow Coma Scale. Which scenario best demonstrates a correct application of this scale? A. The nurse applies pressure to the nail bed, and the client tries to push the nurse's hand away. The nurse scores motor response as "localization of pain" B. The nurse asks the client what day it is and the client says "banana." The nurse score the response as "confused." C. The nurse speaks with client and then the client's eyes open. The nurse scores eye opening as "spontaneous." D. The nurse walks in the room and the client states "Hi honey. How are you?" The nurse scores viral response as "oriented."

Answer: A

The nurse is caring for a client in the immediate postoperative period following an exploratory laparotomy after sustaining a gunshot wound to the abdomen. Which assessment finding is most important for the nurse to report to the health care provider? A. Cold and clammy skin B. SpO2 92% C. HR 108 D. U/O 0.6 mL/kg/hr

Answer: A

The nurse walking through a mall parking lot witnesses the collapse of a child. The child is not breathing and has a pulse of 50/min. After the nurse calls emergency services and delivers rescue breaths for 2 minutes, the child is still not breathing and is pale with a pulse of 52/min. What is the nurse's next action? A. Begin chest compressions B. Continue rescue breathing C. Perform abdominal thrusts D. Retrieve defibrillator

Answer: A

The registered nurse (RN) on an orthopedic unit is orienting a new graduate nurse (GN) assigned to a client with a fractured hip and in Buck's traction. The RN intervenes when the GN performs which action? A. Elevates the HOB 45 degrees B. Holds the weight while the client is repositioned up in bed C. Loosens the Velcro straps when the client reports that the boot is too tight D. Provides the client with a fracture pan for elimination needs

Answer: A

What play behavior would the nurse be most likely to observe in a group of 4-year-old children? A. Children playing and borrowing blocks from each other without directing others B. Children playing and working together to build a castle out of blocks C. Children playing next to each other with blocks, but not interacting D. Children playing with blocks by themselves in separate areas of the room

Answer: A

The emergency nurse is triaging clients. Which report is most concerning and would be given priority for definitive diagnosis and care? A. Abrupt, tearing, moving (upper to lower) back pain and epigastric pain B. Severe lower back pain after lifting heavy boxes C. Sharp calf ache with ambulation that improves with rest D. Unilateral leg swelling with 2+ pitting edema after an airplane trip

Answer: A (These are sx's of an aortic dissection/ TEARING= can be fatal/medical emergency!)

Non bilious vomiting is seen in which condition? A. Hypertrophic pyloric stenosis B. Meckel's diverticulum C. Hirschsprung dz

Answer: A (b/c this is a condition where the pathology is proximal to the pyloric sphincter/ before the stomach and NOT passing through the small intestine to the colon which is the reason for the bilious vomiting occurring is Hirschsprung dz)*

Assessment of a client with a history of stroke reveals that the client understands and follows commands but answers questions with incorrect word choices. The nurse documents the presence of which communication deficit? A. Aphasia B. Apraxia C. Dysarthria D. Dysphagia

Answer: A Aphasia= impaired communication due to a neuro condition. - Receptive aphasia= impaired comprehension of speech and writing (impaired Wernickes area) - Expressive aphasia= impaired speech and writing (impaired Brocas area) Not B= b/c this refers to the loss of ability to perform a learned movement due to neuro impairment. Not C= b/c this refers to weakness of the muscles used for speech. Not D= b/c this refers to difficulty swallowing

Which pediatric respiratory presentation in the emergency department is a priority for nursing care? A. Client with an acute asthma exacerbation but no wheezing B. Client with bronchiolitis with low-grade fever and wheezing C. Client with runny nose with seal-like barking cough D. Cystic fibrosis client with fever and yellow sputum

Answer: A B/c "silent chest" is an ominous sign and an emergency priority. Wheezing sounds are not heard due to lack of airflow in general.

A football player is brought to the emergency department after a helmet-to-helmet collision without loss of consciousness or signs of external trauma. Which clinical finding warrants immediate intervention? A. Hairnet-like effect across vision B. Loss of memory about the collision C. Temporal HA D. Tongue laceration oozing blood

Answer: A B/c this sign is indicative of retinal detachment.

The nurse has just reassessed the condition of a post-op client who was admitted 1 hour ago to the surgical unit. The nurse plans to monitor which parameter most carefully during the next hour? A. Urine output of 20 mL/hr B. T of 37.6 C (99.6 F) C. BP 100/70 D. Serous drainage on the surgical dressing

Answer: A Because Urine output should be at least 30 mL/hr or 0.5-1 mL/kg/hr

*The nurse is caring for a client immediately after removal of the ET tube. The nurse should report which signs IMMEDIATELY if experienced by the client? A. Stridor B. Occasional pink-tinged sputum C. RR 24 D. A few basilar lung crackles on the right

Answer: A Because this high-pitched, coarse sound that is heard over the trachea with the stethoscope indicates airway edema and places the client at risk for airway obstruction.

The nurse is reviewing a surgeon's prescription sheet for a pre-op client that states that the client must be NPO after midnight. The nurse should call the surgeon to clarify that which medication should be given to the client and not withheld? A. Prednisone B. Ferrous sulfate C. Cyclobenzaprine D. Conjugated estrogen

Answer: A Cyclobenzaprine is a skeletal muscle relaxant.

The nurse receives report on 4 clients. Which client should the nurse see first? A. Client with a right-sided ischemic stroke who is confused and is repeatedly getting out of bed w/o assistance B. Client with an asthma exacerbation who was administered albuterol 15 min ago and has a HR of 110 C. Client with DM who has a blood glucose of 290 and has scheduled insulin apart due D. Client with obstructive sleep apnea who is 12 hr post-op and maintaining an SpO2 of 92 on room air

Answer: A Not B= b/c Albuterol is a beta-adrenergic agonist that stimulates the SNS to cause bronchodilation and relieve asthma symptoms. Expected side effects include tachycardia, palpitations, and tremors.

The client has a chest tube for a pneumothorax. While repositioning the client for an x-ray, the tech steps on the tubing and accidentally pulls the chest tube out. The client's O2 sat drops and HR 132; the nurse hears air leaking from the insertion site. What is the nurse's immediate action? A. Apply an occlusive sterile dressing secured on 3 sides B. Apply an occlusive sterile dressing secured on 4 sides C. Assess lung sounds D. Notify HCP

Answer: A Not B= b/c a tension pneumothorax develops when air enters the pleural space but cannot escape. Thats why tape dressing on 3 sides, b/c it permits air to escape on exhalation and inhibits air intake on inspiration.

The nurse is caring for an African American client with disseminated intravascular coagulation. Which locations are best to assess for the presence of petechiae? A. Buccal mucosa and conjunctiva of the eyes B. Nail beds of fingers and toes C. Palms of the hands and soles of the feet D. Skin over the sacrum and behind the heels

Answer: A Not B= b/c the nail bed of the finger is the best location to assess dark-skinned clients for cyanosis. Not C= b/c the palms of the hands and soles of the feet are ideal locations for assessing other skin color changes that may occur in dark-skinned clients such as jaundice

The nurse is caring for a client with suspected Graves disease. Which assessment finding requires priority intervention? A. Agitation and confusion B. Heat intolerance C. Pulse 110; irregular rhythm D. Red and bulging eyes

Answer: A Not C= b/c tachycardia and arrhythmias (A fib) are commonly seen with hyperthyroidism of any cause, including Graves dz

Nausea and vomiting in which client is of greatest concern to the nurse? A. Client post-op ophthalmic surgery B. Client receiving chemo C. Client with Meniere dz D. Client with severe gastroenteritis

Answer: A Not C= b/c this dz affects the inner ear leading to vertigo, N/V are expected manifestations of this dz.

Which client sx supports a pinworm infection dx? A. Anal itching that is worse at night. B. Intestinal bleeding with anemia C. Poor appetite with weight loss D. Red, scaly, blistered rings on skin

Answer: A Pinworm is easily spread by inhaling or swallowing microscopic pinworm eggs, which can be found on contaminated food, drink, toys, and linens. - Treated with anti-parasitic medications

A 3 month old client has stopped breathing. ID the area where the nurse should check the pulse. A. Brachial artery B. Carotid artery C. Femoral artery D. Radial artery

Answer: A Reason for not B= b/c can be difficult to assess due to a child's shorter neck. This pulse is recommended for clients age over 1 year. Reason for not C= may be used for all clients; however, it is often not easily accessible for palpation due to diapers and clothing. Reason for not D= this is used in RESPONSIVE clients age over 1 year. It is not a recommended method of pulse detection in an unresponsive client as a weak or thready pulse is difficult to palpate at this location.

The nurse is caring for a client with scleroderma. Which assessment finding indicates the most serious complication of the disease and requires priority intervention? A. Abrupt-onset HTN and HA B. Blue and cold fingertips C. Dry cough and exertion dyspnea D. Heartburn and difficulty swallowing

Answer: A Scleroderma is an overproduction of collagen that causes tightening and hardening of the skin and connective tissue. - Renal crisis is a threatening complication that causes malignant HTN due to narrowing of the vessels that provide blood to the kidneys

The nurse is caring for a client who had an anterior wall myocardial infarction 2 days ago. The telemetry technician notifies the nurse at 8:30 AM that the client is in ventricular trigeminy. What is the nurse's priority intervention? A. Admin K+ replacement B. Admin the dose of amiodarone C. Attach cardiac defibrillator pads D. Notify the HCP Lab results- Na+ 140 K+ 3.0 Mg2+ 1.8 Cr 1.1

Answer: A Want to do this first before administering the Amiodarone (anti arrhythmic med), because ventricular trigemini (PVCs recurring) are cxd or exacerbated by hypoxia, LYTE IMBALANCES, emotional stress, stimulants, fever, and exercise

A client with advanced kidney disease has serum potassium of 7.1 mEq/L (7.1 mmol/L) and creatinine of 4.5 mg/dL (398 µmol/L). What is the priority prescribed intervention? A. Admin IV 50% dextrose and regular insulin B. Admin IV furosemide C. Admin PO sodium polystyrene sulfonate D. Prep the client for hemodialysis catheter placement

Answer: A Yes A= b/c insulin shifts K+ from the ECF back into the ICF. The dextrose is added to prevent hypoglycemia. Not B or C= b/c furosemide and Kayexalate although do decrease K+ in the blood, they take a lot longer to remove K+ from the body. - Note: Sodium polystyrene sulfonate (Kayexalate) administered by mouth or enema to remove K+ in the body by exchanging Na+ for K+ ions in the intestines to be excreted in feces.

The health care provider has explained the risks and benefits of a planned surgical procedure and asks the nurse to witness the client's signature on the consent form. Which situation would affect the legitimacy of the signature? A. Client asks whether a blood transfusion will be required during surgery B. Client expresses a fear of post-op pain C. Client received a dose of hydrocodone for pain 12 hr ago D. Client wishes to wait to sign the consent until the spouse is present

Answer: A Yes, b/c this indicates an incomplete understanding of risk of the procedure which invalidates the signature.

The unit secretary notifies the nurse that 4 clients called the nurses' station reporting pain. Which client should the nurse assess first? A. Client who had a foot amputation today reporting left shoulder pain radiating down the arm B. Client who has acute pancreatitis reporting severe, continuous, penetrating abdominal pain C. Client who has multiple myeloma reporting deep pelvic pain after walking down the hall D. Client who has sickle cell disease reporting severe pain in the arms and upper back

Answer: A b/c these are ex's of an MI (Pain radiation from the neck, jaw, LEFT shoulder, arms, and epigastrium)

The nurse is caring for a client on a mechanical ventilator. The settings on the ventilator have just been changed, and the standing prescription is to draw arterial blood gases 30 minutes after a ventilator change. In anticipation of this blood draw, what intervention should the nurse implement? A. Avoid suctioning the client B. Pre-oxygenate the client C. Raise the HOB D. Reduce the amount of sedation med

Answer: A b/c suctioning can deplete the client's O2 level and cause inaccurate test results Not B= b/c this can also affect the ABG results. Note: pre-oxygenation should occur prior to suctioning and possibly before position changes

The nurse is caring for a child who had a tonsillectomy. Which of the following are appropriate nursing interventions? Select all that apply. A. Educate parents to expect ear pain and give acetaminophen PRN B. Encourage the child to drink cold liquids thru a straw C. Notify the HCP about frequent, increased swallowing D. Notify the HCP about the development of bad breath E. Use an oral suction device to regularly remove secretions from the back of the throat

Answer: A and C Not B= b/c drinking thru a straw creates suction that causes localized pressure at the back of the throat and can contribute to bleeding. Yes A b/c one of the expected post-op findings include ear pain when swallowing (ex= referred pain from the throat) - Other expected findings include low-grade fever (under 101F) and a superficial infection at the surgical site causing a white, fluid-filled area of exudate in the throat with bad breath*

A nurse is reading a client's tuberculin skin test 48 hours after placement and notes an 11-mm area of induration. The client is a recent immigrant from Nigeria and reports no symptoms. Which actions would be appropriate by the nurse? Select all that apply. A. Ask the client about a history of bacilli Calmette-Guerin vaccine B. Doc the - response in the client's medical record C. Have the client return in a week to receive a second injection D. Obtain a prescription for a CXR from the HCP E. Place the client on droplet precautions and wear a surgical mask during care

Answer: A and D Yes A= b/c this vaccine improves TB resistance in high-risk countries but produces false-+ TB test results. Not C= b/c a client with latent TB doesn't have sx's and unless a CXR or sputum culture is +, the client has only latent TB (not contagious).

A client who has had abdominal surgery complains of feeling as though "something gave away" in the incisional site. The nurse removes the dressing and notes the presence of a loop of bowel protruding through the incision. Which intervention should the nurse take? Select all that apply. A. Contact the surgeon B. Instruct the client to remain quiet C. Prepare the client for wound closure D. Doc the findings and actions taken E. Place a sterile saline dressing and ice pack over the wound F. Place the client in a supine position without a pillow under the head

Answer: A, B, C, and D

The nurse reinforces education about safety modifications in the home for the spouse of a client diagnosed with Alzheimer disease. What instructions should the nurse include? Select all that apply. A. Arrange furniture to allow for free movement B. Keep frequently used items within easy reach C. Lock doors leading to stairwells and outside areas D. Place an identifying symbol on the bathroom door E. Provide a dark room free of shadows for sleeping

Answer: A, B, C, and D

The nursing instructor asks a nursing student to explain the characteristics of the amniotic fluid. The student responds correctly by explaining which as characteristics of amniotic fluid? Select all that apply. A. Allows for fetal movement B. Surrounds, cushions, and protects the fetus C. Maintains body T of the fetus D. Can be used to measure fetal kidney function E. Prevents large particles such as bacteria from passing to the fetus F. Provides an exchange of nutrients and waste products between the mother and fetus

Answer: A, B, C, and D

The nurse cares for a client who is experiencing exophthalmos as a complication of Graves' disease. Which nursing action(s) should be included in the client's plan of care? Select all that apply. A. Admin artificial tears to moisten the conjunctiva B. If eyelids don't close during sleep, lightly tape them shut C. Recommend the use of dark glasses to prevent irritation D. Teach about the importance of smoking cessation E. Teach avoidance of eye movement to prevent further damage

Answer: A, B, C, and D Not E= b/c it is important to teach the client to perform intraocular muscle exercises to maintain flexibility.

A client with chronic bronchitis tell the home health nurse of being exhausted all day due to coughing all night and being unable to sleep. The client can feel thick mucus in the chest and throat. What interventions can the nurse suggest to help mobilize secretions and improve sleep? Select all that apply. A. Increase fluids to at least 8 glasses (2-3 L) of water a day B. Sleep with a cool mist humidifier C. Take prescribed guaifenesin cough med before bedtime D. Use abdominal breathing and huff cough technique at bedtime E. Use pursed-lip breathing during the night

Answer: A, B, C, and D Note: Guaifenesin (Robitussin) is an expectorant that reduces the viscosity of thick secretions by increasing respiratory tract fluid.

A client comes to the emergency department with crushing substernal chest pain. Which interventions should the nurse complete? Select all that apply. A. Admin morphine B. Check BP and HR C. Draw blood specimen D. Obtain a 12-lead ECG E. Position client in the supine position

Answer: A, B, C, and D Yes C= b/c drawing blood will help in ID cardiac markers (like troponin greater than 0.1) indicative of MI/ cardiac damage.

The emergency department nurse assesses an elderly client who was just admitted with a fractured hip after a fall. Which assessment findings would the nurse most likely expect? Select all that apply. A. Ecchymosis over the thigh and hip B. Groin and hip pain with weight bearing C. Internal rotation of the affected extremity D. Muscle spasm around the affected area E. Shortening of the affected extremity

Answer: A, B, D, and E

The nurses on a medical-surgical unit maintain a shared social media page. Which social media posts written by nurses breach client confidentiality? Select all that apply. A. "Im going to private-message everyone a cute story about our sweet client with dementia" B. "It breaks my heart that our paraplegic client was so neglected by her husband" C. "So proud of how well our nurses worked together yesterday, despite how busy we were!" D. "The client in room 5 is + for influenza, so please remember your flu vaccines!" E. "Wash your hands well if you had room 4 this week! Cultures are now + for C diff"

Answer: A, B, D, and E

The nurse prepares to admit a client for worsening cirrhosis who is on the waiting list for a liver transplant. Based on the client's laboratory results, the nurse anticipates which assessment findings? Select all that apply. A. Ascites B. Bruising C. Constipation D. Itching (pruritus) E. Lethargy

Answer: A, B, D, and E A b/c the low serum albumin from the lack of the liver to produce albumin. B b/c the thrombocytopenia (low platelets) causing an elevated PT/INR. D b/c the increased unconjugated bilirubin levels. E b/c the high serum ammonia due to the livers lack to conjugate it into urea to be excreted into the urine.

An intoxicated client not wearing a seatbelt drives into a metal barricade near the entrance to the emergency department. The client's head has hit the windshield, and the client is unconscious. What nurse actions are appropriate? Select all that apply. A. Assess the client for a carotid pulse B. Determine the client's GCS score C. Maintain airway with head-tilt/chin-lift maneuver D. Place a hard cervical collar on the client E. Remove the client from the car onto a backboard

Answer: A, B, D, and E Not C= b/c the nurse should use the jaw-thrust technique to establish the airway.

A nurse reviews the plan of care for a client who has increased intracranial pressure. Which nursing actions should be included? Select all that apply. A. Admin a stool softener B. Dim lights when not providing care C. Elevate head on several pillows D. Maintain body in midline position E. Only perform oral suctioning when necessary

Answer: A, B, D, and E Yes A= b/c this decreases straining w/ defecation. Yes D= b/c keeping head and body midline and avoiding extreme hip or neck flexion is good as not doing so impedes venous drainage Not C= b/c elevating the HOB is preferred over utilizing pillows to elevate head as pillows may flex the neck, decrease venous drainage, and increase ICP.

The nurse is caring for a female client newly diagnosed with epilepsy who has been prescribed phenytoin. Which of the following should the nurse include in client teaching? Select all that apply. A. "Avoid drinking alcoholic beverages" B. "Do not abruptly stop taking your phenytoin" C. "Go to the ED overtime a seizure occurs" D. "Wear an epilepsy medical ID bracelet" E. "You may need to start using a non hormonal birth control method"

Answer: A, B, D, and E Yes E b/c Phenytoin (Dilantin) is an anticonvulsant that may decrease the effectiveness of some meds (Ex= oral contraceptives, warfarin) due to the stimulation of hepatic metabolism. Note: Anticonvulsants should not be stopped abruptly, as this increases the risk of seizure.

What clients will require the nurse to institute contact precautions? A. MRSA B. C. diff C. Pertussis infection D. Vancomycin-resistant Enterococcus E. Influenza F. Scabies

Answer: A, B, D, and F Pertussis and Influenza is spread by droplet.

A client with emphysema arrives at the clinic for a routine follow-up visit. Which manifestations are characteristic of emphysema? Select all that apply. A. Activity intolerance B. Barrel chest C. Hyperresonance on percussion D. Stridor E. Tracheal deviation

Answer: A, B, and C Emphysema is characterized by alveolar wall destruction. Lung tissues lose elasticity (recoil) due to permanently enlarged, "floppy" alveoli. This causes hyperinflation of the lungs (air trapping), manifested by hyperresonance on percussion and prolonged expiration. - Barrel chest manifests from hyperinflation of the lungs ("air trapping")

Which assessment findings should the nurse anticipate in a child with suspected acute otitis media (AOM)? Select all that apply. A. Frequently pulling on the affected ear B. Refusal to eat C. Restlessness and irritability D. Retracted tympanic membranes E. Severe pain with pressure on the tragus

Answer: A, B, and C Not D= b/c membrane would be bulging Not E= b/c this s/s is associated with otitis externa.

A client is at 28 weeks gestation with suspected preeclampsia. Which are potential signs/symptoms related to this syndrome? Select all that apply. A. 2+ pitting pedal edema B. 300 mg/24 hr protein in urine C. Frequent urination D. HA and blurry vision E. Hgb 10 g/dL

Answer: A, B, and D

The nurse assesses a client with Cushing syndrome. Which clinical manifestations should the nurse expect? Select all that apply. A. Hyperglycemia B. HTN C. HypoNa+ D. Truncal obesity E. Wt loss

Answer: A, B, and D

The nurse is caring for an adult client who is in soft wrist restraints. Which nursing actions should be included in the plan of care? Select all that apply. A. Offer fluids, nutrition, and toiling q 2 hours and PRN B. Perform neuromuscular assessment q hr C. Reassess client's continued need for restraints q 12 hr D. Release restraints to perform ROM exercises q 2 hr E. Remove restraints for trial discontinuation every 4 hr

Answer: A, B, and D Not C= b/c nurse should regularly reassess q hr the client's continued need for restraints.

The emergency department nurse is assessing a client brought in after a car accident in which the client's head hit the steering column. Which assessment findings would indicate that the triage nurse should apply spinal immobilization? Select all that apply. A. Breath smells of alcohol B. Client disoriented to place C. Client reports eyes burning D. Hx of multiple sclerosis E. Point tenderness over spine

Answer: A, B, and E

The nurse is caring for a client with multiple renal calculi. Which nursing interventions should be included in the plan of care? Select all that apply. A. Admin analgesics at regularly scheduled intervals B. Encourage fluid intake of up to 3L/day C. Instruct client to stay on bed rest D. Provide massage to the client's flank E. Strain all urine for presence of stones

Answer: A, B, and E Not C= b/c ambulation and frequent mobilization is ENCOURAGED as tolerated to help facilitate the passage of renal calculi

The nurse assesses a client receiving peritoneal dialysis. Which assessment findings are most important for the nurse to report to the health care provider? Select all that apply. A. Cloudy outflow B. Low-grade fever C. Oliguria D. Pruritis E. Tachycardia

Answer: A, B, and E These are the correct answers, b/c cloudy outflow, tachycardia and a low-grade fever are signs of peritonitis (an infection of the peritoneal cavity and a MAJOR concern with peritoneal dialysis). Note: 1. --Bloody fluid can indicate intestinal perforation or that the client may be menstruating -- Brown effluent can indicate fecal contamination from perforation (Both of these above findings must be reported to the HCP) 2. Not D= b/c itching is a common finding in clients with kidney failure and is resultant from the inadequate removal of waste products (ex= urea, calcium-phosphate)

The nurse is caring for a client who received extracorporeal shock wave lithotripsy with ureteral stent placement for treatment of a kidney stone. Which discharge instructions provided by the nurse are appropriate? Select all that apply. A. "Contact your HCP if you develop a fever or chills" B. "Except for using the bathroom, you should stay on bed rest for the next 48 hr" C. "Increase your fluid intake to help flush out the kidney stone fragments" D. "It is common to have some blood in the urine up to 24 hr after this procedure" E. "You may develop some bruising on your back or on the side of your abdomen"

Answer: A, C, D, and E

Which client situation would be classified as an adverse event, requiring the nurse to complete an incident report? Select all that apply. A. CSF sample is sent to the lab labeled as a urine sample B. Client who has a Hgb of 6 refuses recommend blood products C. Nurse does not report K+ result of 6.5 to HCP D. Postpartum client who is post epidural anesthesia falls while ambulating to the bathroom E. Provider prescribes 5,000 units of heparin, nurse gives 1mL (10,000 units/mL) of heparin

Answer: A, C, D, and E

A hospitalized client develops acute hemorrhagic stroke and is transferred to the intensive care unit. Which nursing interventions should be included in the plan of care? Select all that apply. A. Admin PRN stool softeners daily B. Admin scheduled enoxaparin injection C. Implement seizure precautions D. Keep client NPO until swallow screen is performed E. Perform frequent neuro assessments

Answer: A, C, D, and E Not B= b/c anticoagulants are contraindicated in clients with hemorrhagic stroke; the nurse should question any prescriptions that increase risk of bleeding.

A client is diagnosed with right-sided Bell's palsy. What instructions should the nurse give this client for care at home? Select all that apply. A. Apply a patch to the right eye at night B. Avoid driving C. Chew on the left side D. Maintain meticulous oral hygiene E. Use a cane on the left side

Answer: A, C, and D Not B and E= b/c vision, balance, consciousness, and extremity motor function are not impaired with Bell's palsy.

After addressing a group of young adults about sexual health and hygiene, the nurse recognizes that teaching regarding genital warts and the human papillomavirus (HPV) has been effective when hearing which client statements? Select all that apply. A. "Genital warts that have been treated are at risk of recurrence" B. "I should begin Pap testing as soon as I am sexually active" C. "I should receive the HPV vaccine series even if I am already sexually active" D. "Infxn with HPV increases my risk of cervical cancer" E. "Using condoms during sex will eliminate the risk of spreading the virus"

Answer: A, C, and D Not B= b/c Pap testing should be initiated at age 21 regardless of sexual hx.

The nurse plans to administer 0900 meds via NG tube route to a client with an NG tube. The nurse contacts the HCP to clarify which prescriptions that are contraindicated using this route? Select all that apply. A. Enteric-coated Ibuprofen 200mg tab B. Extra-strength Acetaminophen 500mg tab C. Metoprolol extended-release 50-mg tab D. Sulfamethoxazole double-strength 800mg tab E. Tamulosin 0.4mg slow-release capsule

Answer: A, C, and E B/c enteric-coated drugs, slow/extended/ or sustained release drugs should NOT be given this route. Doing this may clog the NG tube or alter the property of the drug leading to increased likelihood of adverse effects.

A healthy 50 y/o client asks the nurse, "What must I do in preparation for my screening colonoscopy? Which statements by the nurse correctly answer the client's question? Select all that apply. A. "No food or drink is allowed 8 hrs prior to the test" B. "Prophylactic antibiotics are taken as prescribed" C. "Smoking must be avoided after midnight" D. "The day prior to the procedure you r diet will be clear liquids" E. "You will drink polyethylene glycol as directed the day before"

Answer: A, D, and E

The parent of a 21 day old male infant reports that the infant is "throwing up a lot." Which assessments should the nurse make to help determine if pyloric stenosis is an issue? Select all that apply. A. Assess the parents feeding technique B. Check for fam hx of gluten enteropathy C. Check of hx of physiological hyperbilirubinemia D. Check if the vomiting is projectile E. Compare current weight to birth weight

Answer: A, D, and E

The nurse is teaching about the importance of dietary fiber at a community health fair. Which health benefits of consuming a fiber-rich diet should the nurse include in the teaching plan? Select all that apply. A. Helps prevent colorectal cancer B. Improves glycemic control C. Promotes weight loss D. Reduces risk of vascular dz E. Regulates BMs

Answer: All of the above

Which of the following are considered in a post-op clear liquid diet? A. Apple/grape fruit juice B. Ginger ale/Sprite C. Jello D. Popsicles E. Fat-free broth F. Tea/ coffee

Answer: All of the above

Which of the following assessment data are indicative of pneumonia or atelectasis? A. Dyspnea and increased RR B. Elevated T C. Crackles D. Chest pain and productive cough

Answer: All of the above

A client had a percutaneous nephrolithotripsy 3 hours ago to remove left renal calculi. Since then, the indwelling urethral catheter has drained 125 mL of urine and the nephrostomy tube has drained 0 mL. The client now reports left flank pain radiating to the left groin along with severe nausea. What is the appropriate nursing intervention? A. Assess the urethral catheter for kinks and obstruction B. Irrigate the nephrostomy tube with sterile NS as prescribed C. Irrigate the urethral catheter with sterile NS as prescribed D. Place the client in the prone position to facilitate urine drainage

Answer: B

A client is receiving an infusion of total parenteral nutrition (TPN) with 20% dextrose through a central line at 75 mL/hr. The nurse responds to the client's IV pump alarm, which indicates that the bag is empty. The new bag is not expected to arrive from the pharmacy for an hour. What is the most appropriate nursing action? A. Hang 0.9% NS until new bag arrives, then increase TPN to 150 mL/hr B. Hang 10% dextrose in water until the new bag arrives, then resume TPN at 75 mL/hr. C. Hang dextran in saline until the new bag arrives, then resume TPN at 75 mL/hr. D. Hang LR until the new bag arrives, then resume TPN at 75 mL/hr.

Answer: B

Based on the nursing assessment progress notes, what is the correct staging of the client's pressure injury? Progress note: Shallow, open area with clean, dark pink wound bed about 1 cm in diameter noted on coccyx. Surrounding area is slightly hard and warm to touch with erythema. Foam dressing clean, dry, and intact. No drainage noted. Enterostomal consult made.________________, RN Options: A. Stage 1 B. Stage 2 C. Stage 3 D. Stage 4

Answer: B

The nurse is caring for a client who has been pronounced brain dead. The client is a registered organ donor. The client's family is voicing concerns about the possibility of disfigurement because they want to have an open casket funeral. How should the nurse respond? A. "If the fam is not in complete agreement about organ donation, we wont be able to proceed" B. "Once the body is dressed, there is no evidence of organ removal. An open casket will be fine" C. "Some organ procurement leaves evidence on the body. You may want to consider a closed casket" D. "Your fam member consented to be an organ donor. You should really honor this wish"

Answer: B

The nurse is caring for a client who was just resuscitated following an out-of-hospital cardiac arrest. The client does not follow commands and remains comatose. What intervention does the nurse anticipate being added to the client's plan of care? A. Assisting the HCP in discussing a DNR order with the family B. Obtaining equipment and cold fluids for induction of therapeutic hypothermia C. Place a small-bore NG tube for enteral nutrition D. Planning for passive ROM exercises to prevent contractures

Answer: B

The nurse is caring for a pediatric client with osteomyelitis. Prior to the nurse administering IV antibiotics, the client's parent states, "We don't believe in antibiotics. Healing comes from within, and medications are toxic to that process." What is the nurse's priority response? A. "Please tell me how meds are toxic to the healing process" B. "Please tell me your understanding of your child's condition" C. What type of healing practices would you prefer for your child?" D. "W/o this med, your child can get worse and could die"

Answer: B

The nurse is planning a client care conference with the parents of a 3-year-old with newly diagnosed type 1 diabetes mellitus. What is the priority outcome for the caregivers? A. Demonstrating adequate coping skills B. Knowing how to keep blood sugars stable C. Understanding how to perform meal planning D. Understanding the need for periodic follow-up visits

Answer: B

The nurse reads a journal article about a study using a new pain management protocol for clients with terminal cancer. What should the nurse first consider in determining whether the protocol is appropriate to implement on the unit? A. Did the study have institutional review board approval? B. Do the characteristics of the sample population match those of the nurse's unit? C. What are the credentials of the study's researcher? D. What was the financial support provided for the study?

Answer: B

The nurse receives report on 4 clients. Which client should the nurse see first? A. Client admitted 12 hr ago with acute asthma exacerbation who needs a dose of IV methylprednisolone B. Client admitted 2 days ago with congestive HF who is reporting SOB and had an extra dose of furosemide prescribed recently C. Client admitted with intestinal obstruction who is reporting abdominal pain and distention and needs an NG tube placement D. Client who had a cardiac valve surgery 8 days ago but was readmitted with a sternal wound infection and needs antibiotics and a dressing change

Answer: B

The nurse witnessed a signed informed consent for an inguinal hernia repair surgery. During the procedure, the surgeon discovers a secondary ventral hernia that also requires repair. Which action should the nurse perform? A. Add the secondary hereinto the consent form that the client signed before the procedure B. Call the client's MPOA to provide consent for the additional procedure C. Document that an additional hernia was found and that it will require surgery at a later time D. Witness an additional consent after both procedures are complete and the client is awake

Answer: B

The student nurse completes a clinical rotation in the emergency department. The instructor knows the student is able to prioritize care appropriately when the student visits which client first? A. 9 y/o crying with pain and swelling of the left ankle after a popping sound while playing soccer B. 29 y/o with neck swelling and increased pain 2 days after thyroidectomy C. 43 y/o with blood glucose of 423, dehydration, and trace ketones in urine D. 72 y/o who is incontinent with acute altered mental status and is yelling at staff

Answer: B

When caring for a client with a left radial artery catheter, which assessment data obtained by the nurse indicates the need to take immediate action? A. Cap refill less than 3 secs B. Left hand cooler than right C. MAP of 65 D. Pressure bag at 300 mmHg

Answer: B

The nurse is performing a medication reconciliation during a clinic visit with a client recently prescribed lithium. Which medication would cause the nurse to be most concerned? A. Acetaminophen B. Hydrochlorothiazide C. Metformin D. Sulfadiazine

Answer: B (*b/c it is a diuretic which once taken will reduce Na+ levels which can lead to toxicity) Not D= b/c sulfa antibiotics do not interact with lithium so it is safe for the client to take.

A nurse in the cardiac intensive care unit receives report on 4 clients. Which client should the nurse assess first? A. Client 2 months post heart transplant with sustained sinus tach at 110 at rest B. Client 3 hr post coronary artery stent placement via femoral approach and reporting severe back pain C. Client receiving IV antibiotics for infective endocarditis with a T of 101.5F D. Client who had coronary bypass graft surgery 3 days ago and has swelling in the leg used for the donor graft

Answer: B (Going through the femoral approach put the client at increased risk for retroperitoneal hemorrhage)

The nurse assesses a client's surgical incision for signs of infection. Which finding by the nurse would be interpreted as a normal finding at the surgical site? A. Red, hard skin B. Serous drainage C. Purulent drainage D. Warm, tender skin

Answer: B (Serous drainage expected after surgery)

A client suffering from bladder prolapse and subsequent stress urinary incontinence has discussed treatment options with the health care provider (HCP). The nurse evaluates that the client understands support pessary use when the client makes which statement? A. "After the pessary is surgically placed, I'll experience bladder discomfort for several weeks" B. "I can remain sexually active while my pessary is in place" C. "I need to schedule weekly appointments to have the pessary removed and replaced" D. "I should report any vaginal discharge to my HCP immediately"

Answer: B A pessary is a vaginal device that provides support for the bladder. (*Indicated for: anterior vaginal wall prolapse [cystocele]) - Clients can remain sexually active while wearing a pessary. - Surgery is not required for pessary placement; clients are able to insert and remove the pessary themselves.

The nurse on the cardiac unit reviews a current rhythm strip from a client who experienced an inferior wall myocardial infarction. What action should the nurse take first? Click on the exhibit button for additional information. A. Doc the rhythm as an expected finding B. Obtain the transcutaneous pacemaker C. Prep to admin adenosine IV D. Review meds the client is receiving

Answer: B A transcutaneous pacemaker is a NONINVASIVE procedure for bradycardia that requires pads (3) to be placed on a clean/dry chest and a machine to dial in rate and amount of electricity to deliver to assist with pacing. Not C= b/c Adenosine is used to tx SVT. Not D= b/c meds should be reviewed as drug toxicity (beta blockers, Digoxin) can cause this type of block. However, this can be done after other interventions.

The clinic nurse is collecting data on a pregnant client in the first trimester. Which finding is most concerning and warrants immediate intervention? A. Client has not been taking prenatal vitamins B. Client is taking lisinopril to control HTN C. Client reports a whitish vaginal discharge D. Client reports mild cramping pain in the lower abdomen

Answer: B ACE inhibitors such as lisinopril, ramipril, enalaril) and angiotensin II receptor blockers (losartan, valsartan, telmisartan) should be avoided in clients who are planning to become pregnant b/c the drugs are TERATOGENIC.

When a client diagnosed with acute urinary retention is emergently catheterized, the nurse should initially assess for which priority manifestation that may occur as a result of the catheterization? A. Dysuria B. HypoTN C. Infection D. Tachycardia

Answer: B Acute urinary retention is best treated with rapid complete bladder decompression. The nurse should carefully assess for hypoTN and bradycardia, which are potential complications.

The nurse is caring for a 7-year-old client diagnosed with nephrotic syndrome who will be discharged soon. Which statement by the parent indicates the need for further teaching? A. "I'll provide a healthy diet without added salt for my child" B. "I'll organize playdates to keep my child's spirits up during relapses" C. "I'll restrict my child's fluids if I notice swelling or rapid weight gain" D. "I'll test for protein in my child's urine every day"

Answer: B B/c in this autoimmune dz there is a loss of immunoglobulins which can cause an increased susceptibility to infection. Caregivers should minimize the risk of infection during relapses by limiting visitors.

A client with an acute head injury cannot accurately identify the sensation felt when the nurse touches the intact skin with a cotton ball or paper clip. The nurse is aware that the deficit reflects injury to which area of the brain? A. Frontal lobe B. Parietal lobe C. Cerebellum D. Occipital lobe E. Temporal lobe F. Brainstem

Answer: B B/c the parietal lobe of the brain integrates somatic and sensory output. Not A= b/c frontal lobe controls higher-order processing such as executive function and personality. Injury results often in behavioral changes. Not E= b/c temporal lobe integrates auditory input and past experiences (memory). Injury to this area leaves clients with the inability to understand verbal or written language. Not D= b/c occipital lobe registers visual images. Injury to this area= loss/ deficit with vision.

The nurse is reviewing client phone messages. Which client should the nurse call back first? A. Client asking where to take the morning dose of phenytoin b4 surgery the next day B. Client taking dabigatran who reports heavier bleeding with her menstrual cycle C. Client taking metronidazole who reports abdominal cramping and diarrhea D. Client who has taken the last dose of insulin glargine and needs a refill

Answer: B Dabigatran is a thrombin inhibitor that causes clients taking it to be at increased risk for bleeding and hemorrhage. Not A= b/c missing a dose of phenytoin (anti seizure med) could precipitate a seizure, but does not take priority over one with active bleeding.

A nurse is caring for a client with unstable angina. After 5 minutes on a nitroglycerin IV infusion, the client reports relief of chest pain but a new dull, throbbing headache. What is the appropriate nursing action? A. Decrease the infusion rate and reassess the client's report of pain B. Doc the finding and administer prescribed acetaminophen C. Notify the HCP and request a CT scan of the head D. Stop the infusion immediately and notify the HCP

Answer: B Nitroglycerin is an abtianginal med that causes potent vasodilation and is used in the tx of acute coronary syndrome (unstable angina, MI). - IV nitroglycerin admin requires continuous cardiac monitoring and frequent BP assessment (ex= every 15 min for the first hr) - HA is an expected side effect from vasodilation of cranial vessels and should decrease with continuing nitroglycerin therapy - As long as the client does not have severe hypoTN (Sbp less than 90), the finding can be documented and the HA can be treated with ASA or acetaminophen

The charge nurse is making client assignments for the oncoming shift. Which client assignment is most appropriate for a nurse who is 10 weeks pregnant? A. Client receiving brachytherapy for endometrial cancer B. Client with an infected surgical wound + for MRSA C. Client with herpes zoster rash on the face and scalp D. Client with pneumonia who recently traveled to a region with the Zika virus

Answer: B Nurses need to wear lead shielding when providing direct care for a client with an radioactive implant.

The nurse obtains a prescription from a HCP to restrain a client and instructs an UAP to apply the safety device to the patient. Which observation of unsafe application of the safety device would indicate that further instruction is required by the UAP? A. Placing a safety knot in the safety device straps B. Safely securing the safety device straps to the side rails C. Applying safety device straps that do not tighten when force is applied against them D. Securing so that 2 fingers can slide easily between the safety device and the client's skin

Answer: B They should be secured to the bed frame not the side rails.

The nurse is reviewing prescriptions for the assigned clients. Which prescription should the nurse question? A. Allopurinol for a client who developed tumor lysis syndrome from chemo for acute leukemia B. Dicyclomine for a client with a history of IBS who develops post-op paralytic ileus C. IV morphine for a client after percutaneous nephrolithtripsy who reports the last BM was 2 days ago D. Levofloxacin for a client with a UTI who has a hx of anaphylaxis to penicillin

Answer: B Yes B= b/c Dicyclomine is an anticholinergic/antispasmodic drug prescribed to manage ex's of intestinal hyper motility with IBS. Therefore it is contraindicated in clients with paralytic ileus as it will decrease intestinal motility and would exacerbate the condition.

A client is being discharged after receiving an implantable cardioverter defibrillator. Which statement by the client indicates that teaching has been effective? A. "Im not worried about the device firing now because I know it wont hurt" B. "I will let my daughter fix my hair until my HCP says I can do it" C. "I will look into public transportation because I wont be able to drive again" D. "I will notify my travel agent that I can no longer travel by plane"

Answer: B b/c you must refrain from LIFTING the affected arm above the shoulder (until approved by the HCP). Not A= Firing may cause pain like a blow to the chest. Not C= Cannot drive a little while after insertion, but driving may be approved by HCP after healing has occurred. Not D= Traveling is not restricted (just know that magnets can interfere with ICD and bring ICD card to airport security when traveling).

The nurse in a clinic is obtaining a developmental history of an 18-month-old during a well-child visit. Which activities should the child be able to perform? Select all that apply. A. Calls self by name B. Goes up stairs while holding a hand C. Stacks 6 blocks in a tower D. Turn 2 pages in a book at a time E. Twists doorknob to open doors

Answer: B and D

The nurse is evaluating how well a client with chronic obstructive pulmonary disease understands the discharge teaching. Which statements made by the client indicate an understanding of the pursed-lip breathing technique? Select all that apply. A. "I exhale for 2 secs through pursed lips" B. "I exhale for 4 secs through pursed lips" C. "I inhale for 2 secs through my mouth" D. "I inhale for 2 secs through my nose, keeping my mouth closed" E. "I inhale for 4 secs through my nose, keeping my mouth closed"

Answer: B and D

The nurse is caring for a client diagnosed with Broca aphasia due to a stroke. Which of the following deficits would the nurse correctly attribute to Broca aphasia? Select all that apply. A. Client coughs and gasps when swallowing food and liquids B. Client is easily frustrated while attempting to speak C. Client is unable to understand speech and is completely nonverbal D. Client misunderstands and inappropriately responds to verbal instruction E. Client's speech is limited to short phrases that require effort

Answer: B and E

The nurse prepares to administer an intermittent bolus enteral feeding to a client via nasogastric tube. Which actions by the nurse are appropriate? Select all that apply. A. Aspirate and discard 50 mL of gastric residual B. Assess the tube placement marking at the naris inception site C. Auscultate bowel sounds prior to feeding D. Elevate the HOB to 45 degrees E. Slow the feeding rate if the client has abdominal cramping

Answer: B, C, D, and E Elevate HOB 30-45 degrees before and 30-60 min after feeding to minimize risk of aspiration.

The nurse prepares to administer potassium chloride to a client through a peripherally inserted IV line. What are the appropriate nursing interventions related to administration of this medication? Select all that apply. A. Admin IV bolus B. Assess IV site frequently C. Assess renal function lab results and urine output D. Place client on a cardiac monitor E. Verify that IV pump infusion is not greater than 10 mEq/hr

Answer: B, C, D, and E For E, the maximum infusion rate of KCl through a PIV is 10 and the max concentration is 40. Higher rates and concentrations require a CVC because it is irritating to the vein.

A client with a severe asthma exacerbation following influenza infection is transferred to the intensive care unit due to rapidly deteriorating respiratory status. Which clinical manifestations support the nurse's assessment of impending respiratory failure? Select all that apply. A. Arterial pH 7.50 B. PaCO2 55 C. PaO2 58 D. Paradoxical breathing E. Restlessness and drowsiness

Answer: B, C, D, and E Not A= b/c pH would be acidic (less than 7.35) Note: - Paradoxical breathing= retractions!

The nurse in the public health clinic is caring for a client with pubic lice. Which statements should the nurse include in the education? Select all that apply. A. "Public lice are only passed through sexual contact" B. "Remove nits from pubic hair with a fine-toothed nit comb" C. "Sexual partners should also receive tx" D. "Wash clothes and linens with hot water" E. "Wash pubic hair with lice tx shampoo"

Answer: B, C, D, and E Not A= b/c pediculosis pubis/ pubic lice ("crabs") may also infest eyelashes, facial hair, and body hair. They may be passed through close contact and sharing of linens. ALL HOUSEHOLD MEMBERS are at risk for developing a public lice infestation and should be screened.

Which components are used in determining the standards of professional nursing practice? Select all that apply. A. Care given with good intention to the best of one's ability B. Clinical practice statements of professional organizations C. Health care institution's policies and procedures D. Nurse Practice Act of the state or province/territory E. Nurse's usual custom and practice

Answer: B, C, and D

A RN is making pre-procedure phone calls to clients scheduled for cardiac pharm nuclear stress testing the following day. Which instructions should the nurse give the clients? Select all that apply. A. Decaffeinated coffee or tea can be consumed B. Do not consume caffeine for 24 hours before the test C. Do not smoke on the day of the test D. Do not take beta blockers on the day of the test E. Take DM meds as usual before the test

Answer: B, C, and D Not A= Decaffeinated beverages still have a little bit of caffeine. Not E= NPO before the test, therefore taking the meds can cause hypoglycemia if taken w/o food. (*No beta blockers or nitrates b/c these meds may mask the symptoms of angina when the stress test is performed)

The nurse is caring for a client following a transsphenoidal hypophysectomy. Which clinical findings would the nurse recognize as signs that the client may be developing diabetes insipidus? Select all that apply. A. Decreased serum Na+ B. Excess oral water intake C. High urine output D. Increased serum osmolality E. Increased urine specific gravity

Answer: B, C, and D Not A= b/c hyperNa+ would occur with DI Not E= b.c its a decreased urine specific gravity (concentration of the urine, the higher the #= the more dehydrated you may be).

The clinic nurse reinforces teaching to a client with systemic lupus erythematosus. Which instructions will the nurse include? Select all that apply. A. Avoid influenza vaccine B. Avoid situations that cause physical and emotional stress C. Avoid sun exposure and UV light when possible D. Notify HCP if you have a fever E. Use antibiotic soap to cleanse skin rashes

Answer: B, C, and D Not A= b/c pneumonia and annual influenza vaccinations are recommended for those with SLE as they are more susceptible to infections

The nurse moves a finger in a horizontal and vertical motion in front of the client's face while directing the client to follow the finger with the eyes. Which cranial nerves is the nurse assessing? Select all that apply. A. II B. III C. IV D. V E. VI

Answer: B, C, and E

The nurse provides teaching for the parents of a 6-year-old diagnosed with nocturnal enuresis. Which of the following instructions with the nurse include? Select all that apply. A. "Allow your child to wear a diaper at bedtime to avoid accidents" B. "Have your child assist with wet linen changes" C. "Prepare a calendar with your child for logging wet and dry nights" D. "Restrict oral fluids to 8 oz with each meal" E. "Wake your child at a specified time each night to void"

Answer: B, C, and E

A client with a T4 spinal cord injury has a severe throbbing headache and appears flushed and diaphoretic. Which priority interventions should the nurse perform? Select all that apply. A. Admin an analgesic PRN B. Determine if there is bladder distention C. Measure the client's BP D. Place the client in Sims' position E. Remove constrictive clothing

Answer: B, C, and E Not A= b/c should administer a meds to decrease BP to assist with tx the HA. Not D= b/c place in HOB elevated 45 or high Fowlers to relieve pressure in the head.

When caring for a client with ulcerative colitis, which nursing activities are appropriate for the registered nurse to delegate to the licensed practical nurse? Select all that apply. A. Admin a blood transfusion B. Admin a prescribed suppository C. Discuss dietary modifications with the dietitian D. Monitor for change in bowel sounds E. Remind the client to track daily weights

Answer: B, D, and E Yes E b/c the LPN can REINFORCE teaching that was already provided by an RN.

The LTC nurse is performing assessments on several of the residents. Which are normal age-related physiological changes the nurse should expect to note? Select all that apply. A. Increased HR B. Decline in visual acuity C. Decreased RR D. Decline in long-term memory E. Increased susceptibility to UTI F. Increased incidence of awakening after sleep onset

Answer: B, E, and F

A client diagnosed with septic shock has an upward-trending glucose level (180-225 mg/dL [10.0-12.5 mmol/L]) requiring control with insulin. The client's spouse asks why insulin is needed as the client is not a diabetic. What is the most appropriate response by the nurse? A. "It is common for critically ill clients to develop type II DM. We give insulin to keep the glucose level under control" B. "The client was diabetic before, but you just didn't know it" C. "The increase in glucose is a normal response to stress by the body. We give insulin to keep the glucose level at 140-180 mg/dl" D. "This increase is common in critically ill clients and affects their ability to fight off infection. We give insulin to keep the glucose level in the normal range (70-110 mg/dL)

Answer: C

A client is brought to the emergency department by emergency medical services with a flaccid right arm and leg and lack of verbal response. The stroke alert team is initiated. The nurse takes which priority action? A. Determine onset of symptoms B. Ensure that the client has 2 large-bore IV lines C. Maintain patent airway D. Prepare for head CT scan

Answer: C

A client returns to the unit after receiving hemodialysis for the first time. The client vomits once, reports headache, and appears restless and disoriented. What is the priority intervention? A. Admin antiHTN that were held prior to dialysis B. Admin PRN ondansetron to relieve N C. Contact the HCP D. Place the client in Trendelenburg position

Answer: C

A client with rheumatoid arthritis (RA) tells the home health nurse, "My fatigue and stiffness are getting worse and I'm having trouble moving around, especially in the morning. What can I do?" Which intervention would be best for the client to perform first? A. Eat a high-calorie carbohydrate breakfast immediately after awakening B. Perform range of motion exercises before getting out of bed C. Take a warm shower or bath immediately after getting out of bed D. Take prescribed NSAID med on awakening

Answer: C

A highly intoxicated client was brought to the emergency department after found lying on the sidewalk. On admission, the client is awake with a pulse of 70/min and blood pressure of 160/80 mm Hg. An hour later, the client is lethargic, pulse is 48/min, and blood pressure is 200/80 mm Hg. Which action does the nurse anticipate taking next? A. Admin atropine for bradycardia B. Admin nifedipine for HTN C. Have CT scan performed to rule out an intracranial bleed D. Perform hourly neuro checks with GCS

Answer: C

A home health nurse is assessing for complications in a client who has been using crutches for 2 weeks. Assessing for which finding is most important? A. Biceps muscle spasm B. Forearm swelling C. Hand and wrist weakness D. Shoulder ROM

Answer: C

The nurse is conducting a prenatal class on the female reproductive system. When a client in the class asks why the fertilized ovum stays in the fallopian tube for 3 days, what is the nurse's best response? A. "It promotes the fertilized ovum's chances of survival" B. "It promotes the fertilized ovum's exposure to estrogen and progesterone" C. "It promotes the fertilized ovum's normal implantation in the top portion of the uterus" D. "It promotes the fertilized ovum's exposure exposure to luteinizing hormone (LH) and follicle-stimulating hormone (FSH)"

Answer: C

The nurse is performing an assessment on a client who is 38 weeks gestation and notes that the FHR is 174. On the basis of this finding, what is the priority nursing action? A. Document the finding B. Check the mothers HR C. Notify the HCP D. Tell the client that the FHR is normal

Answer: C

The student nurse observes the respiratory therapist (RT) preparing to draw an arterial blood gas from the radial artery. The RT performs the Allen's test and the student asks why this test performed before the blood sample is drawn. Which statement made by the RT is most accurate? A. "The Allen's test is done to determine if the cap refill is adequate" B. "The Allen's test is done to determine if the radial pulse is palpable" C. "The Allen's test is done to determine the patency of the ulnar artery" D. "The Allen's test is done to determine the presence of a neuro deficit"

Answer: C Allen test: Step 1- Instruct client to make a fist and occlude both radial and ulnar arteries using firm pressure. Step 2- Instruct the client to open the fist and the palm of the hand will exhibit pallor. Step 3- Release pressure on the ulnar artery and the palm should turn pink in 15 secs indicating potency of the ulnar artery. (*Note: if the Allen's test is +, the ABG can be drawn; if = and the palm does not return to a pink color an alternate site must be used)

A client is brought to the emergency department due to loss of consciousness after binge drinking at a college party and then taking alprazolam. Pulse oximetry shows 87% on room air. Which findings would the nurse expect to assess on an arterial blood gas? A. Metabolic acidosis and hyperventilation B. Metabolic alkalosis and hypoventilation C. Respiratory acidosis and hypoventilation D. Respiratory alkalosis and hyperventilation

Answer: C B/c excessive ETOH and alprazolam (Xanax) causes respiratory depression, which leads to alveolar hypoventilation secondary to the build up of CO2 being expelled= respiratory acidosis.

The nurse is providing care to a client experiencing PTSD following a terrorist attack at the client's place of worship. What is the priority nursing action? A. Acknowledge the clients feelings of anger B. Assess the client's support system C. Encourage the client to talk about the trauma D. Offer the client a PRN sleep med

Answer: C B/c the first step toward resolution of PTSD is the client's readiness to discuss the details of the traumatic event w/o experiencing high levels of anxiety.

The nurse is performing an assessment on an older client who is having difficulty sleeping at night. Which statement by the client indicates the need for further teaching regarding measures to improve sleep? A. "I win 3 times a week" B. "I have stopped smoking cigars" C. "I drink hot chocolate before bedtime" D. "I read for 40 minutes before bedtime"

Answer: C Because chocolate has caffeine.

The nurse is planning care for a client with bipolar disorder and acute mania who is being admitted involuntarily after attempting to run across a five-lane highway. Which intervention is the priority to include in the care plan? A. Assist the client with dressing by giving instructions one at a time B. Collaborate with unit staff to set consistent limits on manipulative behaviors C. Offer high calorie snacks the client can eat while on the move and during tasks D. Secure the client's credit cards to prevent compulsive spending and bankruptcy

Answer: C Clients with BP D/o in acute mania demonstrate hyperactivity and distractibility and may REFUSE to sit still long enough to drink or eat. This characteristic places them at increased risk for inadequate nutritional intake.

A 12-month old is found to have a moderately elevated blood lead level. Which of the following is MOST serious concern for this child? A. GI bleeding B. Growth retardation C. Neurocognitive impairment D. Severe liver injury

Answer: C Lead poisoning particularly affects the neurological system. Therefore elevated levels are dangerous in young children due to immature development of the brain and nervous system.

The risk management nurse is reviewing client records. Which nursing intervention could have contributed to a sentinel event? A. Administered flumazenil to a client who OD on lorazepam B. Administered insulin/dextrose to a client with a K+ of 7.2 C. Administered warfarin to a client with INR of 6 D. Initiated nitroprusside infusion in a client with BP of 210/112

Answer: C Not A= b/c this drug is an appropriate antidote for benzo OD

There has been a major disaster involving a manufacturing plant explosion. The emergency department nurse is sent to triage victims. Which client should the nurse send to the hospital first? A. Client who has partial thickness burns on both hands B. Client who is screaming and has a left lower arm laceration C. Client with a broken, protruding right tibia and gray, pulseless foot D. Client with a gaping head wound and GCS score of 3

Answer: C Not D= b/c a large, open head wound and a GCS of 3 is indicative of SEVER neurological trauma. This client has a poor prognosis regardless of treatment (expectant) and would be the lowest priority.

A pregnant client at 38 weeks gestation is admitted to the labor and delivery unit reporting contractions, severe abdominal pain, and dark vaginal bleeding. What is the nurse's priority action? A. Initiate large-bore (18G) PIV line B. Notify OR staff of emergency C-section C. Palpate abdomen and apply FHR monitor D. Perform vaginal exam to assess cervical dilation

Answer: C Not D= b/c a vaginal exam is not performed in the presence of active bleeding until the possibility of placenta previa (placenta is low in the uterus partially or completely covering the cervical opening; usu presents with painless vaginal bleeding).

The nurse cares for a client with a terminal disease who has an advance directive supporting a do not resuscitate (DNR) code status. The client stops breathing and loses a pulse. The client's adult child states, "I changed my mind. Do whatever you can to save him!" Which intervention is most appropriate at this time? A. Call for help to initiate CPR B. Call the HCP to confirm the DNR status C. Explain the client's wishes to the client's child D. Offer to call the hospital chaplain to provide support

Answer: C Not D= b/c this support should be offered AFTER the client's wishes are explained to the child.

A client has been given instructions about collecting a urine specimen to test creatinine clearance. The client indicates correct understanding of the specimen collection procedure by making which statement? A. "A catheter is placed temporarily then removed after I void" B. "I must provide a midstream sample in a sterile container" C. "I will need to collect all my urine in a container for 24 hours" D. "The first AM specimen is best as it is more concentrated"

Answer: C Note: a creatinine clearance test is to measure glomerular function. This test requires a 24-hr urine (when the test begins, the 1st urine specimen is DISCARDED and time is noted)

A nurse receives change-of-shift report on 4 clients. Which client should the nurse assess first? A. Client who experienced a TIA 2 days ago and is due to receive scheduled ASA B. Client who had a subdural hemorrhage 36 hr ago and is requesting a breakfast tray C. Client with a bowel resection receiving TPN who had 4800mL of u/o the last shift D. Client with a stroke receiving TPA whose GCS changed from 9 to 13

Answer: C TPN has glucose (dextran) a colloid solution as its primary component. RN must monitor TPN clients for s/s of hyperglycemia (polydipsia, polyphagia, polyuria, HA, and blurred vision). If hyperglycemia is not tx immediately= can lead to seizures, coma or death.

The nurse is teaching a group of new parents about oral hygiene for their children. One of the parents asks, "When should I take my child to the dentist?" What would be the best response from the nurse? A. "It is recommended that your child's first dental visit be after age 1" B. "The first visit should be when all of your child's baby teeth have come in" C. "The initial visit should be soon after the child's first tooth appears" D. "Your child will need to be taken to the dentist before starting pre-K"

Answer: C b/c children should have their 1st dental visit within 6 months of the 1st tooth eruption or by their 1st bday. A child's 1st tooth eruption occurs around 6 months of age.

The nurse taught the caregiver of a child with a ventriculoperitoneal (VP) shunt about when to contact the health care provider (HCP). The caregiver shows understanding of the instructions by contacting the HCP about which symptom? A. A T of 99F that occurs during the evening B. The child cannot recall items eaten for lunch the previous day C. The child V after awakening from a nap and 1 hr later D. The VP shunt is palpated along the posterior-lateral portion of the skull

Answer: C b/c vomiting may be a sign of increased ICP.

The nurse is performing an initial assessment on a client diagnosed with Addison's disease. Which assessment findings should the nurse anticipate? Select all that apply. A. Acanthuses nigricans B. Hirsutism C. Hyperpigmented skin D. Truncal obesity E. Weight loss

Answer: C and E Not A= b/c this is a skin condition that occurs with obesity and DM and appears in valet-like patches of darkened, thick skin (think Kate)

The nurse is assessing the functioning of a chest tube drainage system in a client who has just returned from the recovery room following a thoracotomy with wedge resection. Which are the expected assessment findings? Select all that apply. A. Excessive bubbling in the water seal chamber B. Vigorous bubbling in the suction control chamber C. Drainage system maintained below the clients chest D. 50mL of drainage in the drainage collection chamber E. Occlusive dressing in place over the chest tube insertion site F. Fluctuation of water in the tube in the water seal chamber during inhalation and exhalation

Answer: C, D, E, and F

The nurse is developing teaching material for a client diagnosed with ulcerative colitis. The client will receive sulfasalazine. Which of the following instructions are included in the discharge teaching plan? Select all that apply. A. Avoid small, frequent meals B. Can have a cup of coffee with each meal C. Eat a low-residue, high-protein, high-cal diet D. Increase fluid intake to at least 2000 mL/day E. Meds should be continued even after the resolution of symptoms F. Take daily vitamin and mineral supplements

Answer: C, D, E, and F Not A b/c small, frequent meals are encouraged to lessen the amount of fecal material present in the GI tract and to decrease stimulation

The nurse is caring for an adolescent client who just had placement of an external fixation device for long-term stabilization of a fractured tibia. Which interventions are appropriate to include in the client's plan of care? Select all that apply. A. Check the pins q 4 hr and if these loose, turn the bolt clockwise to tighten B. Maintain best rest until the device is removed C. Notify the HCP immediately if drainage or increased pain occurs at the sites D. Perform neuromuscular checks q 2-4 hr for 24 hr E. Perform pin care with 1/2 strength hydrogen peroxide and NS solution q 4 hr

Answer: C, D, and E Not A= b/c the RN should assess pins regularly and notify the HCP if they are loose but should not turn the bolts to tighten. Not B= b/c this device allow for early ambulation with the device in place and increases independence while maintaining bone immobilization and prevents immobility hazards.

A client comes to the emergency department after being assaulted. Imaging studies show a simple fracture of the mandible. The nurse assesses edema of the face and jaw, drooling, and bleeding in the mouth; the client rates pain as a 9 out of 10. What is the priority nursing intervention? A. Admin nasal O2 at 3 L/min B. Admin opioids for pain C. Apply ice pack to face for 20 min each hr D. Suction the mouth and oropharynx

Answer: D

A client with a C3 spinal cord injury has a headache and nausea. The client's blood pressure is 170/100 mm Hg. How should the nurse respond initially? A. Admin PRN analgesic med B. Admin PRN Anti-HTN med C. Lower the HOB D. Palpate the client's bladder

Answer: D

A client with mitral valve dz is experiencing uncontrolled A-fib for 3 days that has been unresponsive to drug therapy. The client is scheduled for electrical cardioversion. What other procedure or test does the nurse anticipate that this client will require? A. Chest X-ray B. Exercise stress test C. Insertion of CVC D. Transesophagela echocardiogram (TEE)

Answer: D

A nurse is caring for a client diagnosed with rheumatoid arthritis (RA). Which assessment finding does the nurse expect to assess? A. Asymmetrical pain in the large weight bearing joints B. Low back pain and stiffness that is worse in the morning C. Pain, swelling, and redness of the great toe D. Symmetrical pain and swelling in the small joints of the hands

Answer: D

The charge nurse on a medical-surgical step-down unit is responsible for making assignments. Which client is most appropriate to assign to a new graduate nurse who is still in orientation? A. 65 y/o client POD day 1 L femoral-popliteal bypass graft surgery with a diminished pedal pulse B. 66 y/o client admitted with HTNsive crisis 2 days ago; BP currently 180/102; reports HA and blurred vision C. 75 y/o client with an ischemic stroke transferred from the ICU 1 hr ago; unresponsive with R-sided paralysis D. 78 y/o client with DM and cellulitis of the L foot; requires frequent dressing changes due to excessive drainage

Answer: D

The emergency department nurse receives a client with extensive injuries to the head and upper back. The nurse will perform what action to allow the best visualization of the airway? A. Head-tilt chin-lift in the supine position on a backboard B. Head-tilt chin-lift in the Trendelenburg position C. Jaw-thrust manuever in semi-Fowler's position D. Jaw-thrust manuever in supine position on a backboard

Answer: D

The nurse has just received a prescription to transfuse a unit of pRBCs for an assigned client. What action should the nurse take next? A. Check a set of VS. B. Order the blood from the blood bank. C. Obtain Y-site blood administration tubing. D. Check to be sure that consent for the transfusion has been signed.

Answer: D

The nurse in the emergency department receives report on 4 clients. Which client should be seen first? A. 5 y/o with an accidental epinephrine auto-injector stick and a HR 124 B. 7 y/o who is crying, has vag lacerations and bruising, and has a HR 118 C. 10 y/o with a large, draining abscess on the L buttock and a T 101.2 F D. 14 y/o who is lethargic after playing a football game and has a T of 104.1 F (40 C)

Answer: D

A client is receiving packed red blood cells intravenously through a double-lumen peripherally inserted central catheter (PICC) line. During the transfusion, the nurse receives a new prescription to begin amphotericin B IVPB. What is the nurse's best action? A. Admin amphotericin B through the unused lumen of the PICC line B. Insert a PIV line to begin infusion of amphotericin B C. Interrupt the blood transfusion to infuse amphotericin B, then resume after infusion D. Wait 1 hr after transfusion finishes before administering amphotericin B

Answer: D Amphotericin B is an anti fungal med used to tx systemic fungal infections. The best action is to complete the blood transfusion and allow 1 hr of observation before initiating amphotericin B b/c then the nurse can distinguish between transfusion-related reactions and adverse effects of amphotericin B.

The nurse is reviewing the medical history of a client who has sustained a right tibia/fibula fracture from a fall. The nurse identifies which finding as most likely to hinder healing? A. BMI of 29.5 B. Family hx of osteoporosis C. Hx of a daily glass of wine D. Peripheral arterial dz

Answer: D B/c bone healing depends on multiple factors including nutrition, adequate circulation, and age. Not B= b/c osteoporosis is not a CURRENT problem for this client, therefore fam hx itself would not directly hinder bone healing.

A 3 month-old infant has irritability, facial edema, a 1-day hx of diarrhea with adequate oral intake, and seizure activity. During assessment, the parents state that they have recently been diluting formula to save money. Which is the most likely cause for the infant's sx? A. HypoNa+ due to diarrhea B. Hypoglycemia due to dilute formula intake C. HypoK+ due to excess GI output D. HYpoNa+ due to water intoxication

Answer: D B/c infants have immature renal systems with a low GFR resulting in decreased ability to excrete excess H2O leading to water intoxication. Common sx's of water intoxication are: irritability, lethargy, and in severe cases, hypothermia and seizure activity.

The nurse in the endocrinology clinic is reviewing phone messages from clients. Which client would be the priority to call first? A. Client with a hx of thyroidectomy who needs a refill for levothyroxine B. Client with Addison dz who is taking corticosteroids and reports new mood swings C. Client with DM who reports blood sugars of 250-300 in the past week D. Client with hyperthyroidism who has a new T of 101.5

Answer: D B/c this client is at risk for developing thyroid storm which is a life-threatening form of hyperthyroidism.

The nurse on the antepartum unit is performing shift assessments of several pregnant clients. Which client assessment is the priority to report to the health care provider? A. Client with gestational DM reporting dysuria B. Client with hyperemesis gravidarum with BP 90/48 C. Client with oligohydramnios and a reactive fetal non stress test D. Client with preeclampsia with 3+ reflexes and 2 beats of clonus

Answer: D Clients with preeclampsia are at risk for developing preeclampsia- associated seizure activity (eclampsia) as a result of increased CNS irritability. - Presence of neuro manifestations (hyperreflexia, clonus/ convulsions) may indicate worsening preeclampsia and can precede seizure activity.

The nurse is inserting a NG tube in an adult client. During the procedure, the client begins to cough and has difficulty breathing. What is the most appropriate action? A. Insert the tube quickly B. Notify the HCP immediately C. Remove the tube and reinsert it when the respiratory distress subsides D. Pull back on the tube and wait unit the respiratory distress subsides

Answer: D It is not necessary to remove the tube completely when respiratory distress occurs.

The nurse is preparing a symptom management teaching plan for a client diagnosed with carpal tunnel syndrome. Which instruction is appropriate to include in the teaching plan? A. Apply elastic compression hose to wrists B. Avoid use of caffeinated or tobacco products C. Perform repetitive hand exercises daily D. Wear a wrist immobilization splint

Answer: D Not A or C= b/c doing either or both of these could worsen the symptoms of CTS by increasing median nerve compression.

A client taking morphine sulfate for acute pain has not voided in 6 hours. The nurse suspects the client has developed urinary retention. What is the priority nursing intervention? A. Ask if the client needs to use the bedpan B. Assess the client's fluid intake C. Assess the client's skin turgor D. Palpate the client's suprapubic area

Answer: D Opioids (ex= morphine sulfate), anticholinergics and TCAs can cause urinary retention. - The RN should assess the suprapubic area first for distention and dull to percussion before proceeding with interventions for urinary retention (doing this first will determine if the client has urinary retention FIRST!)

A nurse in a clinic is talking with a parent about the onset of puberty in boys. What is the first sign of pubertal changes that occurs? A. Appearance of upper lip hair B. Increase in height C. Presence of axillary hair D. Testicular enlargement

Answer: D The first manifestation of puberty and sex maturation is testicular enlargement and scrotal changes. This typically occurs at age 9.5-14. It is followed by the appearance of pubic, axillary, facial, and body hair.

A client arrives in the emergency department with right-sided paralysis and slurred speech. The nurse understands that the client cannot receive thrombolytic therapy due to which reason? A. Client had gallbladder surgery 2 months ago B. Client has experienced loss of the gag reflex C. Client has a platelet count of 130,000/mm3 D. Client has symptoms that started 12 hours earlier

Answer: D Thrombolytic therapy (tissue plasminogen activator) is used to dissolve blood clots and restore perfusion in clients with ischemic stroke. The nurse assesses for contraindications to tPA due to the risk of hemorrhage. Yes D= b/c tPA must be administered within a 3-4.5 hr window from onset of symptoms for full effectiveness.

Juvenile idiopathic arthritis

Associated with risk for becoming reconditioned due to decreased muscle strength and endurance and overall capacity for exercise. - Generally low impact, weight-bearing, and non-weight bearing exercises that involve ROM and stretching to preserve joint mobility and strengthen muscles are best.

A critically ill client receiving vasopressor therapy for hypotension requires continuous blood pressure monitoring via an arterial catheter. The nurse sets up the pressure monitoring system and correctly places the transducer where?

At the phlebostatic axis (external anatomical point on the chest at the level of the atria of the heart) - level of atria at the 4th intercostal space at the midaxillary line

How do Na+ levels affect the renal excretion of lithium?

B/c lithium and Na+ are excreted in a parallel mechanism by the kidney. If Na+ intake is LIMITED or the body is depleted of its normal sodium, then lithium is REABSORBED by the kidneys, increasing the potential for LITHIUM TOXICITY.

Why is a continuous IV solution such as NS required to be administered with a morphine infusion via PCA pump?

B/c the NS is required to keep the vein open (KVO) and flush the PCA medication through the line so that the boluses reach the client.

Why should a patient not take St. John's Wort with an SSRI?

B/c the herbal supplement mimics the action of SSRIs by increasing available serotonin in the brain. Having the two concurrently can lead to serotonin syndrome. Serotonin syndrome= characterized by mental status changes, autonomic dysregulation, and neuromuscular hyperactivity

Obtain blood cultures before or after initiating antibiotic therapy?

Before initiating antibiotic therapy.

Digoxin medication

Blood pressure support and anti arrhythmic. - Positive inotropic drug that improves contractility. - Used in long-term treatment of HF.

Difference between decorticate and decerebrate posturing

Both of these types of posturing are abnormal and occur with some type of brain injury. Decorticate: - "Pts will bring their arms into their CORE" - Flexed posturing indicating damage to the cerebral hemispheres - Arms adducted and flexed - Legs are internally rotated and feet plantar flexed Decerebrate: - "Arms will be extended" - Can indicate damage to the brainstem - WORST type of posturing between the 2 types - Arms adducted, extended, pronated - Legs straight with feet flexed downward

What are dental caries?

CAVITIES (form when bacteria digest carbohydrates in the mouth, producing acids that break down tooth enamel and cause mineral loss.

What is the difference between a PIV, CVC, and a midline?

CVC (or Peripherally inserted central line/ PICC) goes straight to the HEART. Midline goes 1/2 way to the heart. Peripheral IV (PIV) is very short.

What kind of drug is Nifedipine?

Calcium channel blocker to reduce electrical condition of the heart (decreases HR) and reduce workload on the heart.

What kind of drug is Verapamil?

Calcium channel blocker to reduce electrical condition of the heart (decreases HR) and reduce workload on the heart.

What kind of drug is Diltiazem?

Calcium channel blocker to reduce electrical conduction of the heart (decreases HR) and reduce workload on the heart.

Dialysis Disequilibrium Syndrome

Caused by the rapid removal of urea during hemodialysis - Rare but potentially life-threatening complication that can occur in clients during the initial stages of hemodialysis - Can be prevented by slowing the rate/ stopping dialysis - Urea is removed more quickly from the blood than from the brain cells and CSF, creating a conc gradient that can lead to excess fluid in the brain cells and increased ICP. = characteristic manifestations such as N/V, HA, restlessness, change in mental status and seizure activity occur due to increased ICP HCP should be contacted IMMEDIATELY if this is expected.

How long can you leave a PIV inserted for/ how often does it need to be changed?

Change a PIV every 72-96 hours (q 3-4 days)

Rhabdomyolysis

Destruction of striated muscle (caused by trauma, extreme exertion, or drug toxicity; in severe cases renal failure can result) to produce myocytes.

intimate partner violence

Domestic violence or battering between two people in a close relationship

A single transverse crease extending across the palm of the hand, small and low-set ears, flat nose bridge, protruding tongue, and hypotonia are signs of?

Down Syndrome

Why is Methylergonovine contraindicated for clients with high BP who is suspected to have a postpartum hemorrhage?

Due to the primary mechanism of action being vasoconstriction. If administered to a hypertensive client, further BP elevation, seizure, or stroke could occur.

How often do you change a central line dressing?

Every 7 days

Fundal massage is performed after??

Expulsion of the placenta to increase uterine tone and decrease bleeding.

Precipitous labor is...

Extremely rapid labor and delivery. Occurs when the newborn is delivered in 3 hours or less after the onset of contractions.

Fetal tachycardia is defined as

FHR > 160 lasting more than 10 minutes May be an early sign of fetal hypoxia

How will you place a male or female patient when inserting a urinary catheter? Hint: What position and they are different positions

Female: Dorsal recumbent position (supine with legs flexed) - Laying flat on back with knees flexed (not the same as lithotomy position)* Male: Supine position with thighs slightly abducted

What does GTPAL stand for?

G= gravida (# of times pregnant) T= term births (longer than 37 weeks gestation) P= preterm births (before 37 weeks of gestation) A= abortions or miscarriages L= (current) living children

What is insensible fluid loss?

H2O loss through the skin and lungs that we are unaware of losing.

If the bag of IV solution is empty and the nurse is waiting for the delivery of another bag of solution from the pharmacy, a 10% dextrose in water solution should be infused at a prescribed rate to prevent what?

HYPOGLYCEMIA

What is alopecia?

Hair loss

The CAM-ICU tool is used to detect?

If delirium is present.

What is the relationship between concentration of phosphorus and calcium?

If phosphorus is elevated, then there will be reduced calcium.

A neurological check includes assessing what 4 things?

If the patient is oriented to... 1. person 2. place 3. time 4. situation

What is pneumonia?

Inflammation of the alveoli in the lungs caused by an infectious process that may develop 3-5 days post-op as a result of infection, aspiration, or immobility

Encephalitis

Inflammation of the brain

percutaneous kidney biopsy

Insert a needle through skin to obtain a tissue sample that is then used to determine cause of certain kidney diseases. *uncontrolled hypertension is contraindicated! Most surgeries can do below 50k platelets. Only neurosurgery and ocular surgery needs above 100k.

What is myxedema coma?

Life threatening condition that occurs if hypothyroidism is not treated. - Causes decreased LOC that may progress to a comatose state

What is a transesophageal echocardiogram?

Looks at the heart through the esophagus. This is an alternate way to perform an echocardiogram with a specialized probe to be passed into the esophagus. - Allows image and Doppler eval to be recorded. - Need sedation for the procedure.

Paralytic ileus

Obstruction of the intestine due to paralysis of the intestinal muscles. (This is fatal!)

What is a Volkmann contracture? (associated with clients with cast/ MSK injuries)

Occurs as a result of compartment syndrome. - Restricts arterial blood flow resulting in ischemia and tissue damage, wrist contractors, and inability to extend the fingers - This is a medical emergency

Physiologic anorexia

Occurs when the very high metabolic demands of infancy slow down to keep pace with the moderate growth of toddlerhood. - During this phase, toddlers are increasingly picky about their food choices and schedules.

The nurse prepares to draw up regular and NPH insulins into one syringe. Place in order the steps that the nurse should take when mixing the insulins. - Clean the vial tops with alcohol swabs - Draw up NPH insulin - Draw up regular insulin - Inject air into the NPH insulin vial - Inject air into the regular insulin vial

Order- 1. Clean the vial tops with alcohol swabs 2. Inject air into NPH insulin vial 3. Inject air into the regular insulin vial 4. Draw up the regular insulin 5. Draw up the NPH insulin (it goes insulin reg THEN NPH!!*)

Carpal tunnel syndrome

Pain and paresthesia (burning or prickling sensation) of the hand caused by median nerve compression within the carpal tunnel at the wrist. - Nerve compression can occur due to inflam of the tendons; narrowing/compression of the carpal tunnel; or wrist flexion or extension - Wrist immobilization splints reduces pain by preventing flexion or extension and subsequent nerve compression - Clients with CTS may require surgery to permanently relieve symptoms.

Arthritis

Painful inflammation and stiffness of the joints.

To avoid supine hypoTN in the pregnant client, what position should you place her in?

Place the client on her left side to address the aortocaval compression.

Cervical cerclage

Placed to prevent preterm delivery. It is a heavy suture that is placed transvaginally or transabdominally to keep the internal cervial os closed. Placement occurs at 12-14 weeks gestation for clients with a history of cervial insufficiency.

What are the differences between placenta accreta, placenta increta, and placenta percreta?

Placenta accreta: placenta extends a little in the myometrium Placenta increta: placenta extends fully into the myometrium Placenta percreta: placenta extends into myometrium and perimetrium

Which insulin is the only insulin that can be administered IV push?

Regular insulin (short-acting) Note: other insulins such as NPH (intermediate), glargine (long), and apart (rapid acting) are ONLY indicated SubQ route.

What are the 6 Rights?

Right... 1. Patient 2. Time 3. Drug 4. Dose 5. Route 6. Reason

S1 and S2

S1= First heart sound. The beginning of systole. Produced by closure of mitral and tricuspid valves, coincides with carotid artery pulsation. S2= Second heart sound. The end of systole. Produced by closure of aortic and pulmonic valves.

Which position would the nurse place a client recovering from a R modified radical mastectomy who is admitted from the PACU?

Semi-Fowlers position with the affected side's arm on several pillows. (*this will help to promote drainage and prevent venous and lymphatic pooling)

What is in each stage 1-4 of pressure ulcers?

Stage 1: Intact skin with nonbanchable redness (minor sun burn) Stage 2: Partial-thickness skin loss (abrasion, blister, or shallow crater) involving the dermis or epidermis; the wound bed is red or pink and may be shiny or dry Stage 3: Full-thickness skin loss; subcutaneous fat is visible but not tendon, muscle, or bone; tunneling may be present Stage 4: Full-thickness skin loss with visible tendon, muscle, or bone; slough or eschar (scabbing, dead tissue) may be present; undermining and tunneling may be present Unstageable: Full-thickness skin loss; ulcer base is covered by slough and/ or eschar that needs removal to stage

What types of precautions would be implemented for clients with Hepatitis C or HIV positive?

Standard precautions

What is the term for when a seizure lasts greater than 5 mins ongoing OR w/o returning to normal (having several seizures back-to-back w/o subsiding)?

Status Epilepticus (MEDICAL EMERGENCY) - Tx includes Benzodiazepines (Diazepam) b/c they increase the amount of GABA in the brain which will help in reducing the excitation in the brain that is causing the epileptic episode. -- This is also why benzos are used for anxiety as well, b/c they believe that increasing the GABA assists in decreasing the stimulant causing the anxiety.

What to do when placental insufficiency occurs from late of variable decelerations?

Stop oxytocin, happy O2 and notify HCP

What are the "TORCH" infections that pregnant women need to avoid to reduce causing fetal abnormalities?

T= toxoplasmosis O= other (parvovirus, varicella-zoster virus) R= rubella C= cytomegalovirus H= herpes simplex virus

What does a peak expiratory flow rate measure?

The amount of air that can be forcefully exhaled in 1 second. This number indicates the amount of airway obstruction. Ex= After an asthma attack, the PEFR should be higher than it was when the airway was constricted during the attack. Measured in L/min.

Why should a pregnant woman be cautious when exposed to cat feces, ingesting undercooked meat, or contaminated soil with fruits/veggies?

To prevent getting Toxoplasmosis (a parasitic infection caused by Toxoplasma gondii), which may be acquired from exposure to infected cat feces or ingestion of undercooked meat or soil-contaminated fruits/vegetables. - Pregnant clients who contract toxoplasmosis can transfer the infection the to fetus and potentially cause serious fetal harm.

What is oligohydramnios?

Too little amniotic fluid which may lead to umbilical cord compression and fetal compromise. - A reactive non stress test is a reassuring finding.

What is the difference between Topical and Transdermal medication administration routes?

Topical- Has effect AT THE SITE of application. Therefore the patch would need to me placed near the site of the pain to be effective. Ex= Lidocaine Transdermal- Has an effect in areas of the body AWAY from the site of application. Ex= Fentanyl

Inflammatory Bowel Dz's- What are the main differences between ulcerative colitis and Crohn's dz?

Ulcerative Colitis- 1. Affects mucosa and submucosa of the large intestine ONLY 2. Spreads starting at the rectum and is circumferential (covers entire lumen) before moving up the large intestine (into the descending colon). 3. Surgical removal of the large intestine cures the dz. 4. Known to have an autoimmune cx. Crohn's dz- 1. Destruction extends beyond through the entire depth of the intestinal wall AND can cx inflammation and tissue damage ANYWHERE along the GI tract from mouth to anus (but most commonly in the ileum, colon, or both). 2. Destruction is scattered THROUGHOUT the GI tract. 3. Colon removal will not cure the dz. 4. Cx from an disregulated immune-related d/o (overactive immune system from an invading pathogen).

Oxygen delivery devices

• NC: 25-40% 1-6 lpm, hypoxemia/hypercapnia need low levels of 1-2 lpm. Nursing: assess skin, pad tubing around ears • Simple face mask: 50-60% 8-12 lpm, allows room air to enter mask, for short term O2 or emergency like seen in L&D, poorly tolerated • Venturi mask: 24-50% 3-15 lpm, precise concentrations of O2, for COPD, keep NC on hand for meals, do oral/skin care • NRB: 60-100% 6-15 lpm, w/ reservoir bag, one way valve, highest %, sickest pts, close monitoring, bag should deflate slightly when pt inhales and expand when pt exhales


Kaugnay na mga set ng pag-aaral

MIS - Final Exam - Quizzes and Test

View Set

ATI Engage Fundamentals: Vital Signs

View Set

CIT 15 Chapter 12 Technology in Action 16e Fresno City College Fall 2019

View Set

Fluid regulation, water, electrolytes

View Set